You are on page 1of 58

SAMPLE QUESTIONS

A Collective Arrangement by All Editors and Associate Editors

ANESTHESIA . . . . . . . . . . . . . . . . . . . . . . . . . . . . . 2 NEUROSURGERY . . . . . . . . . . . . . . . . . . . . . . . . . . 26
CARDIAC AND VASCULAR SURGERY . . . . . . . 3 OBSTETRICS . . . . . . . . . . . . . . . . . . . . . . . . . . . . . . 27
CARDIOLOGY . . . . . . . . . . . . . . . . . . . . . . . . . . . . . 4 OPHTHALMOLOGY . . . . . . . . . . . . . . . . . . . . . . . . 28
COMMUNITY HEALTH . . . . . . . . . . . . . . . . . . . . . 5 ORTHOPEDICS . . . . . . . . . . . . . . . . . . . . . . . . . . . . 30
DERMATOLOGY . . . . . . . . . . . . . . . . . . . . . . . . . . . 7 OTOLARYNGOLOGY . . . . . . . . . . . . . . . . . . . . . . . 31
EMERGENCY MEDICINE . . . . . . . . . . . . . . . . . . . 9 PEDIATRICS . . . . . . . . . . . . . . . . . . . . . . . . . . . . . . 33
ENDOCRINOLOGY . . . . . . . . . . . . . . . . . . . . . . . . . 11 PHARMACOLOGY . . . . . . . . . . . . . . . . . . . . . . . . . 38
FAMILY MEDICINE . . . . . . . . . . . . . . . . . . . . . . . . 14 PLASTIC SURGERY . . . . . . . . . . . . . . . . . . . . . . . . 39
GASTROENTEROLOGY . . . . . . . . . . . . . . . . . . . . 16 PSYCHIATRY . . . . . . . . . . . . . . . . . . . . . . . . . . . . . . 40
GERIATRIC MEDICINE . . . . . . . . . . . . . . . . . . . . . 18 RESPIROLOGY . . . . . . . . . . . . . . . . . . . . . . . . . . . . 45
GYNECOLOGY . . . . . . . . . . . . . . . . . . . . . . . . . . . . . 19
RHEUMATOLOGY . . . . . . . . . . . . . . . . . . . . . . . . . 46
HEMATOLOGY . . . . . . . . . . . . . . . . . . . . . . . . . . . . 21
UROLOGY . . . . . . . . . . . . . . . . . . . . . . . . . . . . . . . . . 48
INFECTIOUS DISEASES . . . . . . . . . . . . . . . . . . . . 22
ANSWERS TO SAMPLE QUESTIONS . . . . . . . 49
NEPHROLOGY . . . . . . . . . . . . . . . . . . . . . . . . . . . . 23 KEY FEATURE QUESTIONS . . . . . . . . . . . . . . . . 52
NEUROLOGY . . . . . . . . . . . . . . . . . . . . . . . . . . . . . . 25 KEY FEATURE SCORING GUIDE . . . . . . . . . . . 56

MCCQE 2002 Review Notes Sample Questions – 1


ANESTHESIA
1. Which of the following is the proper positioning for 8. The most potent inhalation induction agent that is
intubation? commonly used is:
a) extension of the lower C-spine and extension of the a) enfluane
atlanto-occipital joint b) isoflurane
b) flexion of the lower C-spine and extension of the c) sevoflurane
atlanto-occipital joint d) nitrous oxide (N2O)
c) flexion of the lower C-spine and flexion of the e) halothane
atlanto-occipital joint
d) open the mouth only half way to get a good look at 9. Contraindications to thiopental include all following
the cords EXCEPT:
e) have the patient’s head on a high pillow to facilitate a) cardiac failure
intubation b) porphyria
c) bowel obstruction
2. Which of the following is NOT part of rapid sequence d) potential difficult intubation
induction? e) status asthmaticus
a) patient breathes 100% O2 for 3-5 minutes prior to
induction 10. The following is a depolarizing muscle relaxant?
b) the patient receives bag ventilation immediately a) Rocuronium
following induction b) Mivacurium
c) the Sellick manoeuvre c) Succinylcholine
d) pressure placed on the cricroid cartilage d) Tubocurarine
e) muscle relaxant given e) Pancuronium

3. Which of the following methods is sufficient to 11. Which of the following is FALSE? Epidural analgesia:
determine if the EET has passed through the cords? a) causes uterine relaxation
a) chest movement b) controls blood pressure in preeclampsia
b) absence of abdominal distension c) can cause urinary retention
c) condensation of water vapour in the EET tube d) can contribute to the effect of caval compression
during expiration e) can cause itching
d) CO2 in exhaled gas as measured by capnogragh
e) refilling of reservoir during exhalation 12. All but one of the following increases the likelihood of
regurgitation at the time of induction:
4. Which of the following is CORRECT? a) obesity
a) extracellular fluid volume (ECF) equals 2/3 of total b) anxiety
body water c) upper airway obstruction
b) ECF volume deficit will cause hypotension and d) increased esophageal sphincter pressure
bradycardia e) hiccups
c) ECF volume expansion will cause increased JVP
and S3 13. Which of the following is used in the treatment of
d) moderate dehydration means 10% of the ECF malignant hypertension:
volume is lost a) succinylcholine
e) [Na+] determines ECF volume b) ketamine
c) dantrolene
5. Which of the following is NOT a transfusion reaction? d) tubocurarinee
a) fever e) neostigmine
b) hypercalcemia
c) anaphylaxis 14. Regarding pre-operative investigations, all of the following
d) intravascular hemolysis are true EXCEPT:
e) extravascular hemolysis a) EKG recommended for those > 40 year old
b) pulmonary function test (PFT’s) for those with COPD
6. Which of the following is NOT a side-effect of opioid c) Hb and urinanalysis are required for every surgical
analgesics? patient as indicated in the Public Health Act
a) diarrhea d) hospital policy dictate the indications for tests such as
b) constipation CXR
c) nausea
d) biliary spasm
e) respiratory depression

7. Opioid analgesics:
a) Prevent the release of inflammatory/pain mediators
at the site of injury
b) Block synaptic transmission along the pain pathway
c) Stimulate respiration and increase blood pressure
d) Are rarely associated with tolerance
e) Cause papillary dilation

2 – Sample Questions MCCQE 2002 Review Notes


CARDIAC AND VASCULAR SURGERY
1. Intermittent Claudication is: 4. An aneurysm can be defined as:
a) a reproducible discomfort in exercising muscle a) an artery greater than 5 cm in diameter
groups b) a localized enlargement of a vessel greater than
b) secondary to occlusive arterial disease 1.5X its expected diameter
c) a relatively benign condition when treated with risk c) an artery greater than 3 cm in diameter
factor modification and exercise d) a blood vessel greater than 1.5X its expected
d) may be life-style limiting diameter
e) all of the above e) an out-pouching of the aorta

2. Ischemic rest pain is: 5. The clinical findings of a ruptured abdominal aortic
a) unremitting pain in the most distal portion of the aneurysm include:
affected extremity a) pulsatile abdominal mass
b) a precursor to gangrene and limb-loss b) hypotension
c) associated with pallor on elevation and rubor on c) back pain
dependency d) hematemesis
d) should be treated with aggressive revascularization e) A, B, and C
surgery f) all of the above
e) all of the above

3. An arterial ulcer can be described as:


a) red granulating base. Located over a weight bearing
area. Non-painful
b) red granulating base. Heaped up border with
venous engorgement. Located in the gaiter
distribution
c) whitish, necrotic base. “Punched out” appearance.
No evidence of healing, painful. Located usually on
the distal parts of the extremity
d) all of the above
e) none of the above

MCCQE 2002 Review Notes Sample Questions – 3


CARDIOLOGY
1. All of the following regarding the management of 6. Which of the following provides symptomatic relief but
heart failure are true EXCEPT: has not ben shown to increase longevity in the context of
a) digitalis is indicated in atrial fibrillation CHF:
resulting in heart failure a) ACE inhibitors
b) the use of ß-blockers in acute CHF has been b) loop diuretics
shown to reduce mortality c) beta-blockers
c) calcium channel blockers add no proven survival d) spironolactone
benefit in patients with CHF e) hydralazine and nitrates
d) oral inotropes have a detrimental effect on survival.
e) ACE inhibitors improve survival in 7. The least serious contraindication to Thrombolytic
asymptomatic patients with LVEF < 35% Therapy in acute myocardial infarction is:
a) cerebral hemorrhage
2. Regarding pericardial disease, which of the following is b) active bleeding
INCORRECT? c) acute pericarditis
a) the ECG changes of acute pericarditis include PR d) chronic liver disease
segment depression and ST segment elevation e) aortic dissection
b) autoimmune pericarditis is not associated with
acute MI 8. Which of these is NOT a peripheral sign of infective
c) a prominent “x” descent might be expected in the endocarditis:
JVP waveform of a patient with a pericardial effusion a) Osler's nodes
d) tamponade may result in hypotension, elevated b) splinter hemorrhages
central venous pressure, and pulsus paradoxus c) Janeway lesions
e) a relatively small pericardial effusion may result in d) clubbing
tamponade if it develops rapidly e) palmar erythema

3. Regarding syncope: 9. Which of the following is not one of the major Jone’s
a) the etiology of as many as 50% of cases is never criteria:
identified a) pancarditis
b) vasovagal syncope (the common faint) results from a b) subcutaneous nodules
combination of peripheral vasodilatation and relative c) polyarthritis
bradycardia d) Huntington’s chorea
c) a common cause of syncope is a cortical stroke e) erythema margination
d) the final common pathway of most causes of
syncope is usually generalized cerebral 10. Which of the following is TRUE regarding the JVP:
hypoperfusion a) 3rd degree heart block is associated with cannon
e) cough, micturition, and defecation syncope all result a waves
in part from impaired venous return to the heart b) a positive Kussmaul sign is not correlated with
constrictive pericarditis
4. Which of the following statements is INCORRECT? c) C-V waves are seen in tricuspid stenosis
a) chronic sympathetic nervous system activation d) atrial fibrillatoin does not affect the JVP waveform
causes long-term changes in the myocardium in e) the “y” decent occurs during ventricular systole
heart failure
b) the development of heart failure requires an
initiating myocardial insult
c) dilated cardiomyopathy is a common cause of CHF
d) poor ventricular compliance results in an S3
e) Cheyne-Stokes breathing is frequently seen in
patients with CHF

5. Which of the following statements about valvular cardiac


disease is CORRECT:
a) pulmonary valve disease is a leading cause of
morbidity and mortality
b) rheumatic heart diseases is a minor cause of mitral
stenosis
c) mitral regurgitation leads to LV dilatation
d) radiation of a systolic murmur to the clavicle is not
commonly seen in aortic stenosis
e) mitral valve prolapse leads to a lower JVP

4 – Sample Questions MCCQE 2002 Review Notes


COMMUNITY HEALTH
1. One rationale for universal hepatitis B vaccination in 6. Regarding cocaine abuse:
infants is that: a) cocaine abuse is primarily confined to lower social
a) the vaccine is not effective in preventing infection in classes, and is often associated with opiate abuse
high-risk adults b) cocaine is well absorbed via the oral route
b) no risk factors can be identified in 25% of hepatitis B c) physical dependence is common, and physical
cases withdrawal symptoms are severe
c) hepatitis B infections occur commonly in children d) cocaine may cause toxic psychosis and hypertension
d) the HBV vaccine is contraindicated in at-risk e) cocaine has no current medical indications
pregnant women
e) most chronic carriers contract hepatitis B during early 7. Pneumonia:
childhood a) is the third most common infective cause of death in
the developed world
2. Polyvalent pneumococcal vaccine is NOT b) is very rarely caused by Gram-negative organisms in
RECOMMENDED in patients: elderly patients
a) with sickle cell anemia c) is treated at home in less than 50% of patients
b) with HIV infection d) pleuritic pain is a poor prognostic sign
c) who take prophylactic penicillin e) a poor white cell response is an ominous sign
d) under 2 years of age
e) who are pregnant or considering pregnancy in the 8. A case control study is designed to analyse a suspected
next three months hypothetical association between the development of
mania and migraine treatment. This study:
3. Which one of the following is caused by chronic tobacco a) can derive and estimate of the risk of an individual
smoking? developing the mania as a consequence of migraine
a) increased throat pain during streptococcal infection treatment
b) increased ciliary action b) will demonstrate that any association found is likely
c) increased anti-protease activity to be causal
d) hyperplasia of respiratory epithelium c) will need controls who are chosen at random from the
e) increased risk of ulcerative colitis and endometrial general population
carcinoma d) will necessitate careful follow-up of a group of patient
receiving migraine treatment, and a not treated
4. You are director of occupational health for a corporation control group
that has many employees aged over 45 who smoke one e) will not give biased results if all the patients have
or more packs of cigarettes daily and are at increased been carefully assessed by a senior clinician
risk for lung cancer. What strategy for the early detection
of lung cancer in asymptomatic individuals would you 9. A double blind trial is planned to compare the utility of
recommend? glyburide and metformin in the treatment of diabetes
a) no strategy has been shown to be effective in mellitus. The main reasons for randomising patients are:
reducing mortality a) so that the number of subjects in each group will be
b) chest x-ray and sputum cytology every 6 months for identical
high-risk employees b) so that the two patient groups will have similar
c) annual chest x-ray and sputum cytology for high-risk prognostic features
employees c) so that the statistian will not analyse the data in a
d) annual chest x-rayand sputum cytology for all biased fashion
employees d) so that the investigator does no know in advance
e) annual chest x-ray for all employees what therapy which patient will receive
e) to prevent the clinician knowing which drug the
5. Regarding obesity: patient is taking
a) exercise, diet and behavioural therapy result in
sustained weight loss in 50% of patients 10. Which statement is TRUE?
b) obesity is the leading cause of preventable death in a) the assignment of a gender is a nominal variable
industrialized countries b) histogram is a useful way of showing how a particular
c) the periodic health exam recommends routine diet variable is changing in respect to time
counselling and measuring BMI for all individuals c) the mean of a sample of positive numbers is
d) even a small weight loss is associated with benefit invariably greater than the median value
e) switching from soft drinks sweetened with sugar to d) the mean of a large sample will always increase in
those with artificial sweetners results in fewer size as the size of the sample increase
calories consumed and sustained weight loss for e) approximately 70% of data points are both within one
most patients standard deviation of the mean and larger than the
mean of a normally distributed data set

MCCQE 2002 Review Notes Sample Questions – 5


COMMUNITY HEALTH . . . CONT.

11. Which clinical picture most likely corresponds to a 15. Fetal infection with Hepatitis B is most likely during the:
patient who has ingested “two bottles of aspirin”? a) first trimester
a) arterial thrombosis and hypothermia b) second trimester
b) hyperpnea, tinnitus, and respiratory alkalosis c) third trimester
c) metabolic acidosis, hypokalemia, hyperglycemia d) throughout pregnancy
d) weakness, ataxia, drowsiness e) Hepatitis B is not vertically transmitted
e) facial pallor followed later by hepatic tenderness
16. Appropriate treatment of a neonate born to a Hepatitis
12. Who should be screened for lead poisoning in Canada? B positive mother is:
a) all children with developmental delay a) hepatitis b immune globulin (HBIG) at birth
b) all children living in housing built before 1960 b) HBIG at birth and Hepatitis B vaccine at 2, 4, and 6
c) all children living in housing in high-risk districts months of age
d) children with unexplained symptom complex of c) HBIG and Hepatitis B vaccine at birth, 1, 6 months of
abdominal pain, anorexia, anemia, ataxia, and age
slurred speech d) HBIG at birth and Hepatitis B vaccine at birth, 1 and
e) none of the above 6 months of age
e) hepatitis B vaccine at 2, 4, and 6 months of age
13. Each of the following patients has a 5-mm reaction to a
5TU PPD skin test. Who should receive therapy for 17. With regards to the vertical transmission of Hepatitis B:
tuberculosis? a) there is a less than 1% risk if the mother is
a) an intravenous drug user known to be asymptomatic and HBsAG+
HIV-seronegative b) can be transmitted through the placenta and
b) a Vietnamese refugee through breast milk only
c) a 30-year-old long-term resident of a mental c) risk of vertical transmission is greater than 80% if
institution mother is HBsAg+ and HBcAg+
d) a 40-year-old with diabetes mellitus d) chronic active hepatitis B has no effect on prematurity
e) a 40-year-old female with known HIV infection e) hepatitis B vaccine should not be given to a pregnant
woman
14. A 54-year-old white female who is new to your practice
presents with a viral upper respiratory infection. She 18. Vertical transmission of HIV can be reduced by giving AZT:
insists that three days of penicillin is the only thing that a) during pregnancy
ever cures it. You explain your reasons for not using an b) during labour
antibiotic, but she continues to insist. You should: c) at the delivery
a) treat her symptomatically but do not give antibiotics d) to the neonate
b) refer her to a physician in your community who has e) all of the above
a reputation for frequently using antibiotics
c) have the patient sign a medical release before
giving her a script for gentamycin
d) tell her that if she is not better in three days you will
give her an antibiotic as a compromise

6 – Sample Questions MCCQE 2002 Review Notes


DERMATOLOGY
1. All of the following have been implicated in the 9. Which of the following is NOT a true difference between
pathogenesis of acne vulgaris EXCEPT: erysipelas and cellulitis?
a) androgen stimulated production of sebum Erisypelas Cellulitis
b) Propionibacterium acnes a) Upper dermis Lower dermis
c) consumption of sugars, fats and oils and subcutaneous fat
d) hyperkeratinization of follicle lining b) Well demarcated Poorly demarcated
c) Group A Strep as Dermatophyte
2. Which of the following medications is not indicated for causative agent as causative agent
the treatment of acne vulgaris? d) commonly on face commonly on extremities
a) benzoyl peroxide
b) topical erythromycin 10. Primary syphilis is described by the following EXCEPT:
c) adapalene a) excruciatingly painful buttonlike papule
d) tazarotene b) regional non-tender lymphadenopathy
e) metronidazole c) VDRL negative initially
d) Treated with 2.4 million units benzathine penicillin
3. Which of the following is not a feature of perioral G given intramuscularly
dermatitis?
a) papules and inflammatory plaques 11. Hand-Foot-and-Mouth Disease is:
b) rim of sparing round vermillion border of lips a) common in the elderly
c) predominantly males b) caused by staphylococcus aureus
d) teenagers to mid-adulthood c) caused by Coxsackie A16 virus
a) treated with topical metronidazole d) chronic
e) treated with antibiotic medications
4. You have been referred a 2 month old with atopic
dermatitis. Where are the most likely sites of the 12. Which of the following is not associated with a
dermatitic eruptions in an infant of this age? dermatophytic infection?
a) peripheral fingers and toes a) Tinea capitis
b) flexural creases of elbows and wrists a) Tinea Corporis
c) face and extensor surfaces of limbs a) Condylomata acuminata
d) diaper distribution a) Onychomycosis

5. You have been referred an 80 year old man with a 13. Which of the following typically presents after acute
history of venous insufficiency, who complains of a streptococcal pharyngitis?
constant irritation of his lower legs of 6 months a) atopic dermatitis
duration. The most likely diagnosis is: b) guttate psoriasis
a) seborrheic dermatitis c) pityriasis rosea
b) stasis dermatitis d) plaque psoriasis
c) atopic dermatitis
d) allergic contact dermatitis 14. Which of the following is associated with gluten
intolerance?
6. The cause of “cradle cap” in infants is most commonly: a) Bullous pemphigoid
a) atopic dermatitis b) Pemphigus vulgaris
b) seborrheic dermatitis c) Dermatitis herpetiformis
c) pityriasis rosea d) Porphyria cutanea tarda
d) alopecia areata
15. Which is not a feature of lichen planus?
7. The most common causative agents of Impetigo a) purple
Vulgaris are: b) polygonal
a) pseudomonas and streptococcus c) pruritic
b) fungi d) purulent discharge
c) GABHS and Staphylococcus aureus e) peripheral distribution
d) GABHS and Group B Streptococcus
e) Mycoplasmae 16. All of the following are characteristics of basal cell
carcinoma EXCEPT:
8. Staph aureus group II exfoliating toxin is associated with a) associated with UV radiation
a) bullous impetigo in axillae and groin folds b) pearly nodule with telangiectasia
b) psoriatic knee patches c) located on body surfaces not exposed to sunlight
c) facial atopic dermatitis d) spread by local invasion
d) vitiligo e) age of onset >4 0

MCCQE 2002 Review Notes Sample Questions – 7


DERMATOLOGY . . . CONT.

17. Which is NOT a feature of a cutaneous drug eruption? 20. Malignant melanoma involving the papillary dermis is
a) toxic epidermal necrolysis Clark Level:
b) angioedema a) I
c) phototoxicity b) II
d) hypopigmentation c) III
e) anaphylaxis d) IV
e) V
18. Which is NOT a feature of vitiligo and its management?
a) associated with thryoid disease and diabetes 21. All of the following are treatments for non-scarring
mellitus alopecia EXCEPT:
b) more common in blacks than whites a) spironolactone
c) treatment options include PUVA therapy b) minoxidil
d) destruction of melanocytes c) hair transplantation
e) well-demarcated white macules d) intralesional triamcinalone
e) finasteride
19. All of the following are skin diseases associated with
diabetes mellitus EXCEPT:
a) eruptive xanthomas
b) tinea pedis
c) acanthosis nigricans
d) necrobiosis lipoidica
e) pruritis

8 – Sample Questions MCCQE 2002 Review Notes


EMERGENCY MEDICINE
1. A patient presents with a puncture wound to the hand 7. Which of the following is TRUE regarding hypertensive
that occurred 12 hours ago. He is not certain if he has emergencies?
been vaccinated for tetanus. Which of the following is the a) there is evidence of end organ damage
proper management of this patient with respect to b) renal failure can be both a cause and an effect of
tetanus prophylaxis? hypertensive emergencies
a) tetanus toxoid c) the goal of treatment is to lower the blood pressure
b) tetanus immune globulin to normal within 30-60 minutes
c) both tetanus toixoid and tetanus immune globulin d) fundoscopic findings may include papilledema and
d) no prohylaxis required hemorrhages
e) all of the above
2. A 34 year-old otherwise healthy woman presents in the
trauma room after being involved in a motor vehicle 8. In a head trauma patient the most important feature on
collision. She is strongly suspected to have internal history is:
bleeding. Her heart rate is 130, blood pressure 85/50, a) seizure
and respiratory rate is 40. Which of the following are b) loss of consciousness
likely TRUE? c) headache
a) she has lost < 1,000 cc of blood d) nausea and vomiting
b) she will have normal urine output
c) fluid replacement should consist of crystalloid and 9. According the Ottawa Ankle Rules, an ankle radiographic
blood series is required in all of the following situation EXCEPT:
d) she has lost > 40% of her blood volume a) bony tenderness to palpation along the posterior
edge of the lateral maleolus
3. A patient presents to the emergency department. On b) bony tenderness to palpation along the posterior
examination he requires a sternal rub to open his eyes. edge of the medial malleolus
He is muttering incomprehensively and withdraws his c) bruising over either malleolus
hand when a painful stimulus is applied. His Glascow d) inability to weight bear both immediately
Coma Scale score is: following the injury and in the emergency
a) 7 department
b) 8
c) 9 10. Which of the following is NOT true with respect to
d) 10 hypothermia?
a) chest compressions should be deferred up to
4. With respect to question 3, the most appropriate 1 minute until it is certain that the patient is
management of this patient’s airway is: pulseless as they can precipitate ventricular
a) none tachycardia
b) oropharyngeal airway b) pupils become fixed and dilated at core body
c) nasopharyngeal airway temperature below 25ºC
d) endotracheal intubation c) ventricular fibrillation becomes a risk at core
e) cricothyroidotomy body temperatures < 30ºC
d) the patient should be rewarmed slowly to
5. The anticholinergic and sympathomimetic toxidromes avoid an afterdrop
shareall of the following features EXCEPT:
a) hyperthermia 11. Which of the following is FALSE regarding adult patients
b) maydriasis presenting to the emergency department following a
c) tachycardia sexual assault?
d) diaphoresis a) they will often present with chief complaints other
than sexual assault
6. Which of the following is NOT true regarding the b) physicians are legally obligated to report the
management of asthma patients in the emergency room: assault to the police even if the victim does not
a) patients unable to speak with an O2 sat < 90% wish it reported
should be intubated c) the patient should have folllow up with an MD at
b) peak flow meters are the best way to assess a rape crisis centre within 24 hours
response to therapy d) pregnancy and sexually transmitted disease
c) patient education about the proper use of puffers prophylaxis should be offered if appropriate
is an important aspect of treatment
d) anti-cholinegics are the first line agents in the
treatment of mild asthma
e) a silent chest on auscultaton indicates an
emergency and requires immediate treatment

MCCQE 2002 Review Notes Sample Questions – 9


EMERGENCY MEDICINE . . . CONT.

12. Which of the following is NOT true regarding allergy and 14. Which of the following toxins is NOT a cause of an
anaphylaxis? increased anion gap metabolic acidosis?
a) they are IgE mediated immune response to a) methanol
antigens b) ethylene glycol
b) epinephrine is the first line agent for the treatment c) salicylates
of minor skin eruptions to topical agents d) iron
c) signs and symptoms of an allergic reaction vary d) benzodiazepines
from minor cutaneous eruptions to cardiovascular
collapse, bronchospasm and laryngeal edema 15. Which of the following toxins is NOT matched to its
d) the ABC’s are the first step in the treatment of specific treatment?
any allergic reaction a) acetaminophen - N-acetylcysteine
b) carbon monoxide - hyperbaric oxygen
13. Which of the following is FALSE with respect to chest c) heroin - naloxone
pain? d) diazepam - bicarbonate
a) all patients presenting with chest pain should e) methanol - ethanol
receive an ECG
b) a normal ECG and normal cardiac enzymes rule
an ischemic event
c) a careful history is essential in determining the
etiology of the chest pain
d) reproducible chest pain on palpation of the chest
wall does not rule out an acute MI

10 – Sample Questions MCCQE 2002 Review Notes


ENDOCRINOLOGY
1. A patient complains of a non-tender mass over the 5. A 58 year-old man with a past history of a
thyroid region on the left side of her neck. Concerned parathyroidectomy for primary hyperparathyroidism is
about a thyroid disorder, you order the appropriate now in your office complaining of headaches worse in the
investigations. The results are as follows: AM (made worse by a small MVA he credits to a loss of
TSH: 6.0 peripheral vision). You plan to:
Free T4: 20.2 a) send to the Emergency Department for an
Thyroid antibodies: none immediate CT head
RAIU: No “hot” spots seen b) check his calcium to ensure there’s no remaining
parathyroid tissue
The next investigation(s) you choose to do are: c) check for a pheochromocytoma (which you know
a) watch and wait for 3-6 months causes headaches) because you are concerned he
b) FNA has MEN I syndrome
c) surgical biopsy d) check for a homonymous hemianopia because you
d) trial of L-thyroxine therapy for 6 months are worried about a pituitary tumour
e) none of the above e) check for a bitemporal hemianopia because you are
worried about a pituitary tumour
2. An 8 year-old boy is brought to the office because his
mother is concerned he is entering puberty already. 6. Which of the following is not associated with thyroid
You examine him and note the beginnings of facial hair, disease?
axillary hair and Tanner stage 2 external genitalia. a) dermatitis herpetiformis
Choose the set of investigations you initially want to do: b) urticaria
a) CBC, lytes, testosterone, bone age, CT head c) porphyria cutanea tarda
b) FSH, LH, testosterone, lytes, bone age, DHEA-S d) vitiligo
c) FSH, LH, testosterone, cortisol, DHEA-S, e) alopecia areata
11-OH progesterone, bone age
d) lytes, testosterone, DHEA-S, 17-OH progesterone, 7. In the treatment of Type I Diabetes, which of the
cortisol, bone age following is TRUE?
a) Sulfonylureas are useful as an adjunctive therapy to
3. A 2 month-old boy has a Ca of 1.80 (corrected) after an insulin
assessment for FTT. Mother informs you she has been b) Most patients are adequately controlled with one
consistently breastfeeding without trouble as type of insulin (non-mixed) only
corroborated by a visiting nurse. What is at the top of c) Once diagnosed with Type I DM, patients must
your differential? immediately be assessed for retinopathy
a) malabsorption d) During periods of illness or infection, patients may
b) pseudohypoparathyroidism require additional insulin
c) mother didn’t supplement with DiVisol e) The most common initial presentation is visual
(Vit D supplement) disturbance
d) DiGeorge syndrome
8. Secondary causes of hyperlipidemia include all of the
4. The “triple bolus” test of pituitary function works by a following EXCEPT:
rapid succession of IV constituents as follows: a) obesity
a) insulin – hypoglycemia mediated rise in b) hyperthyroidism
GH and ACTH c) diabetes
LHRH – rise in LH and FSH d) nephritic syndrome
TRH – rise in TSH and PRL e) glucocorticoids
b) CRH – rise in GH and ACTH
LHRH – rise in LH and FSH 9. A 63-year-old woman has had a gradually enlarging goiter
TRH – rise in TSH and PRL for at least 10 years. She has no symptoms of
c) estrogen – rise in LH, drop in FSH and PRL hyperthyroidism. On physical examination, both thyroid
insulin – rise in GH and ACTH lobes are irregular, firm and nontender. Her serum
TRH – rise in TSH thyroxine (T4) level is 120 nmol/L (normal range is
d) cosyntropin – rise in GH and ACTH 0.51 - 142 nmol/L) and triiodothyronine (T3) -resin uptake
GHRH – rise in LH and FSH is normal at 30%. The patient is treated with 50 ug of T3
TRH – rise in TSH and PRL daily in an attempt to decrease the size of the goiter.
Several weeks later she develops weakness,
palpitations, insomnia, and a tremor; she has lost 2.3 kg
(5 lb). The size of the thyroid gland is unchanged; her
serum T4 level is now 115 nmol/L. These findings
indicate that the patient:
a) has developed Graves' disease
b) has painless thyroiditis
c) has autonomous endogenous thyroid function
d) is taking more T3 than was prescribed
e) has had a hemorrhage into a thyroid nodule

MCCQE 2002 Review Notes Sample Questions – 11


ENDOCRINOLOGY . . . CONT.

10. In deciding upon the ideal dosage of replacement 14. All of the following statements correctly pair a useful
therapy for hypothyroidism, each of the following factors medication with its mechanisms of action EXCEPT
is considered EXCEPT: a) Propylthiouracil (PTU) blocks the coupling reaction in
a) subjective patient response T4 synthesis
b) findings on physical examination b) Methimazole (MMU) reduces peripheral conversion of
c) response of the radioactive iodine uptake T4 to T3
d) plasma TSH level c) Radioactive iodine destroys follicular cells in the
e) level of free thyroid hormone measured in the serum thyroid
d) Propranolol blocks the sympathetic components of
11. An asymptomatic postmenopausal 54-year-old woman thyrotoxicosis
sustained a slight concussion in an automobile accident. e) Prednisone may relieve the mechanical exophthalmos
X-rays of the skull show an enlarged sella turcica. and opthalmoplegia by reducing inflammation.
Computed tomography (CT scan) of the skull shows no
density within the sella turcica. This patient most likely 15. All of the following laboratory test results are consistent
has: with the clinical picture EXCEPT:
a) A familial syndrome associated with hypercalcemia a) decreased T3 resin uptake
and pheochromocytoma b) decreased TSH response to a TRH challenge
b) Pigmentation of skin, buccal mucosa, and pressure c) decreased serum TSH
areas d) increased serum T4 concentration
c) Elevated serum calcitonin levels e) positive test for circulating antibodies against the
d) Normal endocrine function TSH receptor
e) Hypogonadism with hyposomnia
16. Which of the following disorders is NOT associated with
12. A 32-year-old diabetic did not take his usual dose of osteoporosis?
30 units of lente insulin and 5 units of regular insulin a) pheochromocytoma
because he had an "upset stomach with vomiting". The b) prolactinoma
next day he was admitted to hospital. His plasma c) Cushing’s syndrome
glucose was 29.9 mmol/L and the urine was strongly d) Kleinfelter’s syndrome
positive for ketones. The serum bicarbonate was e) Graves’ disease.
6 mEq/L, arterial pH was 7.05 and serum potassium was
5.4 mEq/L. Which one the following statements is most 17. A 41-year-old man has been diagnosed with
likely to be TRUE? panhypopituitarism. Which of the following is not part of
a) The predominant ketone body in the serum is the appropriate management of this patient?
acetoacetate a) L-thyroxine
b) The elevated serum potassium level reflects total b) hydrocortisone
body potassium in this patient c) fludrocortisone
c) The plasma glucagon level will be increased d) testosterone
d) The serum phosphate concentration will rise during e) Medic Alert bracelet
insulin therapy
e) The patient followed the correct course by 18. While not always the presenting complaint, the earliest
withholding insulin in the absence of food intake symptom/sign of a pituitary tumour is usually:
a) visual disturbance (e.g. bitemporal hemianopsia)
For 13, 14, and 15. b) gonadal insufficiency
A 32-year-old woman visits her physician because of agita- c) headache
tion, weight loss, and inability to sleep. When questioned d) extraocular muscle paresis
further, she reveals an increased appetite, and an increased e) diabetes insipidus
frequency of bowel movements. Previously, she had regular
menstrual periods, but now they are less frequent and 19. An elderly diabetic woman with mild renal insufficiency
irregular. During the physical examination, the physician presents to the emergency room with confusion. Blood
notes that her skin is warm and moist and that she has a fine glucose was 1.5 mmol/L. She is treated with glucose IV
tremor of the fingers, hyperreflexia, and lid lag. The woman for 24 hours, discharged and instructed not to take any
has moderately severe exophthalmos, and her upward gaze medications. 6 hours after discharge, she is brought back
seems weak and uncoordinated. to the emergency department in a coma. Blood glucose
is now 1 mmol/L. Which of the following hypoglycemic
13. Which one of the following disease processes is most agents is the patient most likely using?
likely manifesting itself? a) repaglinide
a) A thyroid adenoma that is secreting thyroxine b) acarbose
b) Inappropriate hypothalamic secretion of TRH c) metformin
c) Graves’ disease d) rosiglitazone
d) Hashimoto’s disease e) glyburide
e) Sick euthyroid syndrome

12 – Sample Questions MCCQE 2002 Review Notes


ENDOCRINOLOGY . . . CONT.

20. A 47-year-old man is admitted to hospital to be 22. A 35-year-old man gives a history of paroxysmal
evaluated for hypertension and hypokalemia. After the hypertension, headache, diaphoresis and palpitations.
patient has been restricted to a 120 mEq/day Na diet for His father has hypertension and hypercalcemia and a
1 week, his plasma aldosterone twice the normal level. paternal aunt died from a thyroid cancer. The physician
Which is the next MOST appropriate step? would be most likely to measure serum levels of:
a) fludrocortisone suppression test a) calcitonin, calcium, and glucose
b) cosyntropin (ACTH) stimulation test b) glucagon, insulin, and cholesterol
c) measurement of plasma renin activity c) thyroid hormone, adrenomedullin, and phosphorus
d) CT adrenals d) CRH, gastrin, and renin
e) treat with spironolactone

21. Which of the following is considered diagnostic of


diabetes:
a) random plasma glucose level of 10 mmol/L
b) fasting plasma glucose level of 6.8 mmol/L
c) on OGTT, 1 hour plasma glucose level > 11.1
d) on OGTT, 2 hour plasma glucose level > 7.8
e) presence of polyuria and polydipsia and plasma
glucose level of 12 mmol/L

MCCQE 2002 Review Notes Sample Questions – 13


FAMILY MEDICINE
1. A 28 year old sexually active woman in a stable 6. A 65 year-old newly-diagnosed hypertensive male is
relationship for 2 years comes to your office for an about to begin anti-hypertensive medication. Which of
annual check-up. Which of the following is NOT a the following should you NOT prescribe as first-line
recommended screening tool in the Periodic Health therapy:
Exam? a) angiotensin-converting enzyme inhibitor
a) counseling re: sun exposure b) long acting dihydropyridine
b) pap smear c) beta-adrenergic antagonist
c) breast self examination d) low dose thiazide diuretic
d) counseling re: folic acid supplementation e) high dose hydrochlorothiazide
e) counseling re: bicycle helmet use
7. A 32 year old man presents with a 2 week history of
2. A 26 year old woman reports suffering from moderately persistent cough and generally feeling unwell. On
severe unilateral headaches approximately twice a examination, T= 37.7 , HR= 80, BP = 120/70, RR = 16.
month for the past year. The headaches are pulsating in Which of the following is INAPPROPRIATE treatment?
quality with associated nausea and photophobia. a) symptomatic antipyretics
Physical examination is normal. Which of the following is b) antitussives
NOT appropriate abortive treatment for an acute attack? c) bronchodilator bid
a) NSAIDS d) tetracycline 250 mg qid
b) sumatriptan e) rest and fluids
c) DHE
d) amitryptiline 8. A 40 year old smoker presents with a 3 day history of
e) ergotamine rhinorrhea, cough, and headache. No vital sign
abnormalities present. Which of the following is the
3. A 63 year old man with a 10 year history of type 2 best management?
diabetes is screened for microalbuminuria. a) educate patient about duration of symptoms, suggest
Albumin:creatinine ratio is 2.6. Which of the following is hydration, analgesics and dextromethorphan prn
the appropriate first step in management? b) educate patient about duration of symptoms and
a) repeat albumin:creatinine in one year have her return to clinic in 2 days
b) repeat albumin:creatinine test twice over the next c) order CXR
3 months and if abnormal, get 24 hour urine for d) order sputum culture
creatinine clearance e) educate patient about smoke exposure and risk of
c) referral to nephrologist URTI and prescribe erythromycin x 10 days
d) adjust dosages of oral hypoglycemic medications
e) reduce dietary protein 9. Which of the following is CORRECT regarding depression?
a) The lifetime risk of major depressive disorder is 55%
4. A 38 year old businesswoman and mother of two children for women
presents with a 3 month history of fatigue. Past medical b) Early treatment of major depressive disorder does
history is remarkable for GDM in her last pregnancy. not improve outcomes
Pallor noted, otherwise normal physical exam. Which of c) Risk of recurrence after 3 episodes of major
the following investigations is NOT indicated at this time? depressive disorder is 90%
a) appropriate assessment for anxiety and depression d) Most patients will need at least two years of
b) appropriate assessment of current life stressors, pharmacological treatment
past trauma and abuse e) Most primary care patients with depression should
c) CBC be referred for psychiatric consultation
d) serum glucose
e) BUN, creatinine 10. When prescribing the oral contraceptive pill (OCP), which
of the following should be done:
5. The target LDL-C for a very high risk patient (10 year risk a) Inform patients that the OCP protects against STDs
CAD > 30%, history of cardiovascular disease, or b) Perform breast, abdominal and pelvic exams and
diabetes) is: assess blood pressure
a) < 2.5 mmol/L c) Warn patients that their risk of dysmenorrhea is
b) < 3.0 mmol/L increased
c) < 3.5 mmol/L d) Perform a complete physical exam after taking a
d) < 4.0 mmol/L thorough sexual history
e) < 5.0 mmol/L e) Perform a Beta-HCG test prior to prescribing OCP

14 – Sample Questions MCCQE 2002 Review Notes


FAMILY MEDICINE . . . CONT.

11. Which of the following statements regarding pharyngitis 13. Acceptable methods of assessing for obesity include the
is/are TRUE: following but:
a) The most common etiologic agent is viral a) BMI
b) Bacterial causes for sore throats occur predominately b) daily calorie intake
in pre-school aged children c) waist-hip ratio
c) Delaying treatment for Group A ß-hemolytic d) percent body fat
Streptococcal pharyngitis by 48 hours does not
increase the risk of rheumatic fever 14. Which of the following is characteristic of alcohol
d) a) and c) dependence:
e) all of the above a) 23-year-old male college student who consumes an
average of 23 drinks per week and exhibits no
12. In the context of low back pain, x-ray imaging should be withdrawal symptoms
ordered in these cases except: b) 65-year-old recently retired engineer who binge
a) 25-year-old male i.v. drug user with 3 week history of drinks two times a month
low back pain c) 43-year-old married female, recently fired, and who
b) 64-year old female with 48-hour history of back pain has been in 2 MVAs involving alcohol this past year
and numbness in the right leg extending to the knee d) 35-year-old male former AA member who has not
c) Previously healthy 37-year-old male with 3 month drunk in 10 years
history of low back pain
d) 56-year-old woman with well controlled hypertension
and 4 week history of low back pain
e) 18-year-old construction worker with 1 week history
of low back pain

MCCQE 2002 Review Notes Sample Questions – 15


GASTROENTEROLOGY
1. Pseudomembranous colitis is usually caused by toxins of 8. A diagnosis of Irritable bowel sydrome is based on:
which organism? a) Rome criteria which emphasizes negative features of
a) Staph aureus the disease
b) Clostridium difficile b) Rome criteria which emphasizes positvie features of
c) Clostridium perfringens the disease
d) Clostridium botulinum c) a diagnosis of not an exclusion
e) Bacteroides fragilis d) all of the above

2. A 13 year old presents with increased serum bilirubin 9. Diagnosis of celiac sprue includes all of the following
and vague nonspecific complaints. Labs show increase in EXCEPT:
unconjugated bilirubin with normal liver function tests a) evidence of malabsorption
and negative hepatitis screens. There is no evidence of b) abnormal small bowel biopsy
liver abnormality or hemolysis. The most likely diagnosis c) improvement in sign and symptoms with a
is? gluten- containing diet.
a) Gilbert’s syndrome d) positive anti-endomysial antibody.
b) biliary atresia
c) hepatoma 10. You suspect a patient with a fat malabsorption syndrome.
d) cholecystitis Which symptom would you NOT expect?
e) cholangiocarcinoma a) night blindness
b) metabolic bone disease
3. After noticing his wife avidly eats clay, starch, and ice, her c) bleeding disorder
husband brings her to the ER. History reveals she is a d) lactose intolerance
vegetarian, and in her second trimester of pregnancy.
The most likely disease afflicting this woman is? 11. A 12-year-old boy is jaundiced and has an enlarged liver
a) megaloblastic anemia on palpation. Lab values show increased ALT,
b) hepatolenticular disease increased AST, increased conjugated bilirubin.
c) gastric carcinoma Kayser-Fleisher rings are noticed on the slit-lamp.
d) pica A diagnosis of Wilson’s disease is made. All of the
e) hypermethioninemia following are true EXCEPT:
a) the primary defect is a mutation in ceruloplasmin
4. The ecchymosis of the flank seen in patients with acute b) the patient likely has increased levels of serum free
pancreatitis is named after? copper and liver copper and increased urinary copper
a) Trousseau with penicillamine challenge
b) Goodpasture c) Wilson’s disease is inherited in an autosomal recessive
c) Grey-Turner pattern
d) Cullen d) patients can demonstrate increased serum
e) Wernicke-Korsakoff ceruloplasmin
e) in normal patients, more than 90% of serum copper
5. The best test for a massive lover GI bleed is? circulates bound to ceruloplasmin
a) colonoscopy
b) angiography 12. All of the following can be causes of conjugated
c) radiolabelled RBC scan hyperbilirubinemia EXCEPT:
d) barium enema a) Wilson’s disease
e) fecal occult blood a) Criggler-Najjar syndrome I
a) primary biliary cirrhosis
6. An elderly woman presents with blood per rectum with a a) gallstones
3 day history of severe ab pain, anemia, hypotension a) viral hepatitis
and CAD. The most likely diagnosis is?
a) ulcerative colitis 13. The most common cause of a significant
b) crohn’s disease upper GI bleed is:
c) irritable bowel disease a) Mallory Weiss tear
d) ischemic bowel (colitis) a) angiodysplasia
e) infectious diarrhea a) peptic ulcer
a) dieulafoy lesion
7. All are causes of splenomegaly EXCEPT: a) antritis
a) infectious mononucleosis
b) thalassemia minor 14. The medical treatment to be of most benefit in the
c) congestive heart failure therapy of both variceal and non-variceal
d) sarcoidosis upper GI bleed is:
e) diabetes a) octreotide
b) omprazole
c) ice water lavage
d) vasopressin
e) ranitidine

16 – Sample Questions MCCQE 2002 Review Notes


GASTROENTEROLOGY . . . CONT.

15. A 50 year old man was admitted for a bleeding duodenal 16. Which one of the following statements regarding liver
ulcer 2 months ago. He was discharged from hospital failure is TRUE?
with an appropriate course of H. Pylori eradication a) a liver biopsy is a not required with a good history of
treatment. He has since completed the therapy and is alcoholism
currently asymptomatic. Which of the following is the b) encephalopathy may be reversible
BEST non-invasive test to confirm eradition of H. Pylori: c) SBP (spontaneous bacterial peritonitis) is usually a
a) urea breath test result of blunt abdominal trauma
b) endoscopy and histology d) coagulopathy is a result of poor diet in most patients
c) IgG serology with cirrhosis
d) skin testing e) the size a varix is proportional to the degree of
portal hypertension

MCCQE 2002 Review Notes Sample Questions – 17


GERIATRIC MEDICINE
1. Which of the following is NOT a cause of urinary 4. The functional assessment of IADLs in the elderly
incontinence? include all of the following EXCEPT:
a) diabetes a) food preparation
b) coagulopathies b) ability to climb stairs
c) restricted mobility c) driving
d) UTIs d) managing finances
e) prostatic disease e) shopping

2. Which of the following physiological changes affect 5. Which of the following is NOT a cause of failure to thrive in
pharmacokinetics in the elderly? the elderly:
a) increased body albumin a) alopecia
b) decreased body fat b) poor dentition
c) increased GFR c) depression
d) decreased hepatic blood flow d) neglect
e) increased hepatic phase I reactions e) dementia

3. Which of the following statements is TRUE:


a) The ratio of males to females over 85 in Canada
is 2:1
b) Accidents are the most common cause of death in
Canadians over 65
c) During normal bereavement, symptoms of
depression should last for up to one year
d) all of the above
e) none of the above

18 – Sample Questions MCCQE 2002 Review Notes


GYNECOLOGY
1. Which of the following is not consistent with PCOD? 6. Regarding condylomata/genital warts which of the
a) high testosterone following is/are TRUE? (Multiple answers)
b) low LH a) spread can be prevented using condoms
c) average age of presentation 15-35 years b) infection can be latent, only detectable with DNA
d) low or normal FSH hybridization
e) hirsutism c) some viral serotypes are associated with an
increased incidence of cervical CA
2. Which of the following is the most common site of d) removal of lesions is proven to decrease recurrence
occurrence for endometriosis? e) lesions present during pregnancy is an absolute
a) broad ligament indication for C/S
b) uterosacral ligaments f) treatment is through either physical or chemical
c) ovary modalities
d) rectosigmoid colon
e) appendix 7. Which of the following is correct regarding the presentation
f) lung and treatment of genital herpes? (Multiple answers)
a) inguinal lymphadenopathy is present with first
3. Which of the following are diagnostic characteristics of infection
endometriosis? (Multiple answers) b) recurrent infections are usually less severe and less
a) constant pelvic pain frequent
b) chocolate cysts c) presentation is four to six months after exposure
c) deep dyspareunia d) initial presentation is as a painless chancre on vulva
d) blueberry spots e) diagnosis is made on viral culture
e) cyclic dysmenorrhea f) treatment with acyclovir is curative
f) infertility g) outbreak is usually preceded by a tingling, burning
g) bladder symptoms prodrome
h) bowel symptoms
i) powder burn lesions 8. Which of the following are absolutely necessary for the
diagnosis of PID? (Multiple answers)
4. Which of the following are considered risk factors for a) temperature > 39.0 C
ectopic pregnancy? (Multiple answers) b) adnexal tenderness
a) alcohol consumption c) cervical motion tenderness
b) previous PID d) elevated white count
c) use of IUD e) positive culture for N. gonorrhea, or C. Trachomatis
d) appendectomy f) elevated ESR
e) previous ectopic pregnancy g) lower abdominal pain
f) use of assisted reproductive techniques
g) grand multiparity 9. Regarding PAP smear screening protocol, which of the
h) endometriosis following are recommended? (Multiple answers)
a) should be performed yearly from onset of sexual
5. Which of the following are considered predisposing activity until age 69
factors for candidiasis? (Multiple answers) b) requires only an endocervical cell sampling
a) diabetes c) is equally effective in identifying both SCC and
b) pregnancy adenocarcinoma
c) HIV d) a patient can be released from screening after two
d) organ transplant recipient consecutive negative tests
e) BCP e) after three negative tests, screening intervals can be
f) use of a diaphragm increased up to 3 years
g) antibiotic therapy f) an inadequate sample requires a repeat PAP within
h) hypertention 3 months
i) cigarette smoking

MCCQE 2002 Review Notes Sample Questions – 19


GYNECOLOGY . . . CONT.

10. Which of the following are consistent with the clinical 13. A 20 year old G0 woman presents to your office with a
presentation of fibroids? (Multiple answers) 4-month history of amenorrhea. She had previously
a) menorrhagia normal and regular cycles since age 12 years. Your initial
b) abnormal bleeding pattern evaluation of this problem includes:
c) abdominal heaviness a) Thorough history and physical examination
d) increased abdominal girth b) A beta-hCG test
e) amenorrhea c) TSH and prolactin levels
f) infertility d) All of the above
g) abdominal pain e) none of the above
h) difficulty emptying bladder
i) difficulty defecating 14. Absolute contraindications to HRT include:
a) undiagnosed vaginal bleeding
11. Non-contraceptive benefits of oral contraception pills b) known or suspected cancer of the ovary
include: c) acute renal disease
a) Reduced benign breast disease and ovarian cysts d) obesity
b) Reduced dysmenorrhea e) none of the above
c) Reduced anemia
d) Reduced risk of ovarian carcinoma 15. Which of the following condition(s) are associated with
e) All of the above Polycystic Ovarian Syndrome?
a) obesity
12. Regarding infertility: b) hirsutism
a) It is defined as failure to conceive after one year of c) insulin resistance
regular unprotected intercourse d) acanthosis nigricans
b) It occurs in approximately 50-60% of couples e) All of the above
c) Most cases are due to unknown factors
d) A normal sperm count from semen analysis is
> 5 million sperm/mL
e) A hysterosalpingogram plays no role in potential
evaluation of infertility

20 – Sample Questions MCCQE 2002 Review Notes


HEMATOLOGY
1. You would expect to find an increased bleeding time in 9. All of the following may be seen in aplastic anemia
all of the following conditions EXCEPT: EXCEPT:
a) Osler-Weber-Rendu a) Anemia
b) HSP b) Thrombocytopenia
c) ITP c) Leukopenia
d) Celiac disease d) Splenomegaly
e) Splenomegaly e) increase frequency of infections

2. All of the following characterize chronic ITP EXCEPT: 10. The cause of beta thalasemia minor is:
a) onset usually following recent viral infection a) Increase HbA2 production
b) spontaneous remission uncommon b) Reduced B chain production
c) peak age of onset 20-40 years c) Increased HbF production
d) occurs more frequently in women d) No B chain production
e) insidious onset of bleed e) Increased B chain production

3. Which of the following coagulation factors is NOT 11. Metabolic disturbances that may be seen in acute
effected by chronic liver disease myelogenous leukemia include:
a) factor VII a) high uric acid, high calcium, high magnesium
b) factor V b) high uric acid, low calcium, low magnesium
c) fibrinogen c) high uric acid, high calcium, high phosphate
d) factor VIII d) high uric acid, low calcium, low phosphate
e) factor IX e) low uric acid, high calcium, high phosphate

4. What disorder results in prolonged bleeding time and 12. Clinical features of chronic myelogenous leukemia include
decreased factor VIII coagulation activity? all EXCEPT:
a) Hemophilia A a) left upper quadrant pain and fullness
b) Hemophilia B b) pruritus
c) von Willebrand’s disease c) peptic ulcers
d) Vitamin K deficiency d) priapism
e) None of the above e) jaundice

5. A young girl with thalassemia is undergoing a blood 13. The MOST COMMON presentation of essential
transfusion and suddenly develops back pain, fever and thrombocythemia is:
becomes tachypneic. The likely diagnosis is: a) weight loss, fever
a) Delayed hemolytic transfusion reaction b) bleeding
b) Febrile non-hemolytic transfusion recaction c) thrombosis
c) Acute hemolytic transfusion reaction d) asymptomatic
d) Circulatory overload e) splenomegaly
e) Graft vs. host disease
14. A 22-year old male student presents with enlarged cervical
6. Which of the following is not accurate for iron stores: lymph nodes that become painful after alcohol
a) Serum Fe consumption. The only remarkable feature on history is an
b) Transferrin 8 lb weight loss over 2 months. The most likely diagnosis
c) Ferritin would be:
d) Hemosiderin a) ALL
e) TIBC b) Hodgkin’s lymphoma
c) Non-Hodgkin’s lymphoma
7. The most common cause of cold antibodies other than d) CLL
idiopathic is: e) multiple myeloma
a) Drug induced
b) Routine detection 15. Possible treatments for multiple myeloma or its
c) Secondary to infection complications include all EXCEPT:
d) Secondary to lymphoproliferative disorder a) corticosteroids
e) Autoimmune disease b) bisphosphonates
c) radiation therapy for bone lesions
8. Which of the following blood smear results are most d) renal transplant
likely to indicate G6PD deficiency? e) plasmapheresis
a) Rouleaux
b) Howell-Jolly bodies
c) Basophilic stipling
d) Heinz bodies
e) Tear drop cells

MCCQE 2002 Review Notes Sample Questions – 21


INFECTIOUS DISEASES
1. Group A Strep is least likely to cause which of the 10. If a patient comes to your office complaining of chest pain
following complications: and a cough producing a current jelly-like sputum, which
a) scarlet fever organism would be the most likely cause of his/her
b) necrotizing faciitis pneumonia?
c) impetigo a) Streptococcus pneumoniae
d) subacute bacterial endocarditis b) Haemophilus influenzae
e) glomerulonephritis c) Pseudomons aeruginosa
d) Influenza virus
2. Septra is used in AIDS patients to prevent which e) Klebsiella pneumoniae
opportunistic organism?
a) Pneumocystis carinii 11. Which organ in the human body is the most sensitive to
b) M. tuberculosis disseminated intravascular coagulation in a case of
c) S. pneumoniae meningitis?
d) CMV a) thyroid
e) Cryptococcus b) adrenal
c) lateral ventricle of the brain
3. Severe bloody, afebrile diarrhea is associated with what d) spleen
bacterial infection? e) aorta
a) Salmonella infection
b) Enteroinvasive E.coli infection 12. The Negri body is a pathogonomiic sign of:
c) Enterohemorrhagic E. coli infection a) acute bacterial meningitis
d) Enterotoxigenic E. coli infection b) arbovirus encephalitis
e) Giardiasis c) rabies
d) polio
4. Human Herpes Virus 6 is associated with what disease? e) chronic meningitis
a) Infectious mononucleosis
b) CMV infection
c) Herpes Simplex infection 13. If untreated, this infection can lead to cholangiocarcinoma.
d) Roseola a) Schistosomiasis
e) Chicken Pox b) Clonorchis sinesis
c) Strongyloidosis
5. Cat’s scratch disease is caused by what organism? d) Trypanosoma
a) Leishmania spp. e) E. coli
b) Bartonella henselae
c) Treponema pallidum 14. A 10 year old girl from El Salvador is brought to the
d) Toxoplasmosa gondii
e) Ancylostoma braziliensei pediatric outpatient clinic. On the growth charts, she is
< 1 percentile. A CBC reveals a profound anemia. She has
6. Predictors of HIV progression include the following moderate edema of the lower extremities and face. Stool
EXCEPT: is sent for ova and parasites (O&P), and the lab reports the
a) CD4 cell count presence of helminth eggs. The most likely parasite
b) plasma HIV RNA levels at set point compatible with her clinical presentation is?
c) onset of HIV related symptoms a) hookworm
d) age at initial infection b) Strongyloidosis
e) method of transmission (i.e. anal intercourse versus c) pinworm
vaginal intercourse)
d) pork tapeworm
7. Protozoa are associated with all of the following except
a) unicellular organism 15. A 48 year old make who received a bone marrow
b) produce larvae transplant 6 months previously for chronic myelogenous
c) do not cause eosinophilia leukemia is admitted with wedge-shaped infiltrates on
d) indefinite lifespan chest x-ray. He is experiencing severe respiratory difficulty
e) mulitply within host and is coughing up blood. Despite aggressive treatment,
he dies shortly after admission. At autopsy, pulmonary
8. Inwhich disease would Donovan bodies be present? infarcts are found. The pulmonary vessels contain fungal
a) genital warts
b) lymphogranuloma venereum hyphae with acute angle branching. The most likely
c) syphilis diagnosis in infection with:
d) granuloma inguinale a) Asperigillus spp.
e) chancroid b) Candida spp.
c) Blastomycosis
9. Which disease is NOT caused by Chlamydia sp.? d) Pneumocystis carinii
a) lymphogranuloma venereum
b) pelvic inflammatory disease
c) granuloma inguinale
d) inclusion conjunctivitis
e) nongonococcal urethritis

22 – Sample Questions MCCQE 2002 Review Notes


NEPHROLOGY
1. A 63 year old woman with a long history of chronic renal 5. Which of the following is the LEAST APPROPRIATE
failure is seen because of fatigue. She gets tired after indication for dialysis in the setting of acute renal failure?
walking 4 blocks, but is comfortable at rest. Her only a) extracellular fluid volume overload unresponsive to
medication is a calcium channel blocker for diuretics
hypertension. There is no history suggestive of bleeding. b) hyperkalemia unresponsive to medical attempts to
Her physical examination reveals pallor but no other lower the plasma potassium level
abnormalities. Lab investigations reveal that a c) plasma creatinine greater than 300 umol/L
Hgb = 85 g/L, MCV = 88 fL, ferritin = 210 ug/L, in an otherwise asymptomatic patient
vitamin B12 = 210 pmol/L, a normal RBC folate, and d) uremic pericarditis
serum creatinine of 379 umol/L. Two months ago, her e) uremic encephalopathy with seizures
Hgb level was 90 g/L. The MOST APPROPRIATE
THERAPY for her anemia AT THIS POINT is: 6. The COMMONEST cause of acute renal failure in
a) administration of erythropoietin hospitalized patients is:
b) dialysis a) acute crescentic glomerulonephritis
c) renal transplantation b) acute tubular necrosis (ATN)
d) vitamin B12 injections c) cyclosporine nephropathy
e) oral iron d) HIV nephropathy
e) obstructive uropathy
2. A 22 year old woman with a history of type 1 diabetes
mellitus is brought to the emergency room in coma. 7. The COMMONEST cause of death in dialysis and renal
Blood tests indicate a wide anion gap metabolic transplant patients is:
acidosis. If her metabolic acidosis is due solely to a) cardiovascular disease
diabetic ketoacidosis, which of the following findings is b) HIV-associated complications from repeated
NOT consistent: infusions of blood products
a) blood glucose increased c) infection
b) serum ketones increased d) neoplasia
c) serum osmolar gap increased e) suicide
d) extracellular fluid volume reduced
e) urine dip positive for ketones 8. A 21 year old woman is referred for edema of the ankles
that developed 3 weeks earlier. Physical examination
3. A patient in acute renal failure comes to the emergency shows moderate bilateral ankle edema. The chest was
room. His serum potassium value is 8.0 mM. Of the clear and heart sounds were normal. Jugular venous
following ECG changes listed below, which ONE is NOT pressure was 1 cm above the sternal angle. Urinalysis
consistent with hyperkalemia? shows 3+ protein and occasional red and white blood
a) wide QRS complexes cells. Plasma creatinine was normal and plasma albumin
b) peaked T waves was reduced at 36 g/L.
c) peaked P waves Which of the following is the LEAST likely renal
d) ventricular fibrillation diagnosis?
e) prolonged PR interval a) acute tubular necrosis (ATN)
b) membranous glomerulonephritis
4. A chronic dialysis patient who passes no urine has missed c) minimal change glomerulonephritis
3 hemodialysis treatments in the last week and comes into d) lupus nephritis
the emergency room with a serum potassium value of e) post-infectious glomerulonephritis
8.5 mmol/L. An ECG shows cardiac changes consistent with
severe hyperkalemia. Blood glucose is 4.5 mmol/L. Which 9. A 66 year old man is referred to you for hypertension
ONE of the following would be the best action to take: which was not noted until this year, despite yearly
physical examinations for many years. Apart from recent
a) Give the patient a prescription for an oral potassium headaches he has no complaints and was not taking any
exchange resin and arrange dialysis within 24 hours medications. Blood pressure in your office was
b) Give insulin and glucose intravenously 175/100 mm Hg with a pulse of 80/min. Examination of
c) Give insulin and calcium gluconate intravenously and the optic fundi showed arteriolar narrowing. Bilateral
inhaled salbutamol femoral bruits were present. Urinalysis showed 1+
d) Give insulin, calcium gluconate and glucose protein and granular casts. Plasma creatinine was
intravenously and arrange for urgent dialysis 210 umol/L. The MOST LIKELY diagnosis is:
e) Give inhaled salbutamol and larger doses of a) atheromatous emboli
intravenous furosemide b) bilateral renal artery stenosis
c) essential hypertension
d) pheochromocytoma
e) proliferative glomerulonephritis

MCCQE 2002 Review Notes Sample Questions – 23


NEPHROLOGY . . . CONT.

10. A 36 year old man presents with ankle edema and an 13. Treatment of hypertension caused by bilateral renal
elevated jugular venous pressure. Plasma creatinine is artery stenosis should NOT include which of the
350 umol/L. Past history reveals a polyarthritis for following?
6 months treated with ibuprofen and allergic rhinitis. a) weight reduction if obese
Physical exam reveals several active joints in both hands b) drug therapy with angiotensin-converting enzyme
and an itchy skin rash over the trunk. BP = 170/100 mm Hg inhibitors
and he has moderate peripheral edema. Urinalysis shows c) percutaneous transluminal renal angioplasty
a trace of protein, trace blood and on microscopy many d) surgical renal revascularization
white blood cells and white blood cell casts are seen. e) sodium restriction
The MOST LIKELY diagnosis is:
a) atheromatous emboli 14. Hyperkalemia is reduced by each of the following
b) interstitial nephritis EXCEPT:
c) lupus nephritis a) administration of calcium gluconate intravenously
d) membranous glomerulonephritis b) administration of glucose, insulin and sodium
e) renal artery stenosis bicarbonate intravenously
c) oral administration of potassium-exchange resins
11. Acute diffuse proliferative glomerulonephritis is usually with sorbitol
accompanied by each of the following findings EXCEPT: d) hemodialysis with low potassium dialysate
a) red blood cell casts on microscopic urinalysis
b) proteinuria 15. The anemia of chronic renal failure is usually due to which
c) pigmented casts on microscopic urinalysis of the following?
d) blood clots in the urine a) malabsorption of iron
e) none of the above b) vitamin B12 deficiency
c) blood loss
12. The anion gap is increased in metabolic acidosis d) decreased erythropoietin production
associated with each of the following EXCEPT: e) associated inflammatory conditions
a) diabetic ketoacidosis
b) renal tubular acidosis
c) acute tubular necrosis
d) ethylene glycol intoxication
e) lactic acidosis

24 – Sample Questions MCCQE 2002 Review Notes


NEUROLOGY
1. 32 year old woman with right-hand pain waking her up at 8. Choose the ONE CORRECT statement. A lesion of the left
night. Physical exam reveals mild weakness of right cerebellum could cause?
thumb adduction and some thenar wasting. What is the a) akinesia
most likely diagnosis? b) right arm and leg ataxia
a) cervical radiculopathy c) left arm and leg weakness
b) carpal tunnel syndrome d) spasticity of the right arm and leg
c) tendinits e) dysmetria on finger to nose testing of the left arm
d) syringomyelia
9. The following statements concerning the pathophysiology
2. 35 year old woman with one week history of bilateral leg of Parkinson’s disease are correct, EXCEPT:
weakness. Physical exam reveals moderate leg weakness, a) symptoms appear after there has been a substantial
loss of the pigmented neurons of the substantia nigra
mild arm weakness, and decreased reflexes. What is the
b) the internal segment of the globus is the major outflow
most likely diagnosis? center of the basal ganglia
a) Lyme disease c) the globus pallidus interna is normally excitatory
b) Parkinson's disease to the thalamus
c) Multiple Sclerosis (MS) d) a pallidotomy, a surgical lesion placed in the globus
d) Guillian-Barré syndrome pallidus is helpful in some patients with Parkinson’s
disease
3. Patient with mild left-sided hearing loss and absent left e) dopaminergic neurons of the substantia nigra project
corneal reflex. Where is the lesion? to the putamen
a) left cerebellar pontine angle
b) lateral medulla 10. Choose the ONE CORRECT statement regarding Wernicke’s
c) left CN V aphasia:
d) left cochlea a) patients with Wernicke’s aphasia have dysprosodic
speech
4. 75 year old man with sudden onset of severe leg b) weakness of the face and arm are commonly found in
weakness. Physical exam reveals bilateral leg weakness, patients with Wernicke’s aphasia
loss of pain and temperature sensation with intact c) the speech of patients with Wernicke’s aphasia is fluent
vibration and positon sense in both legs. What is the with normal or slightly increased speed
d) patients with Wernicke’s aphasia almost universally
likely diagnosis?
fail to understand the commance “take off your
a) anterior cerebral artery territory stroke
glasses”
b) lumbar radiculopathy e) patients with Wernicke’s aphasia cannot write but
c) anterior spinal artery occlusion they are still able to read
d) diabetic polyneuropathy
11. All of the following statements regarding aphasia are
5. Which of the following is NOT consistent with correct, EXCEPT:
amyotrophic lateral sclerosis (ALS)? a) the left hemisphere is dominant for language even in
a) distal arm and leg weakness many left handed patients
b) coexistence of both upper and motor neuron signs b) patients with Broca’s aphasia often have upper motor
c) slowed motor nerve conduction velocities weakness of the right arm and face
d) fasciculations c) patients with Broca’s Conduction, and Wernicke’s
aphasias all have difficulty with repetition
6. 52 year old woman with incontinence and “dizzy spells.” d) both Wernicke’s and Broca’s areas are supplied by the
Physical exam reveals rigidity, bradykinesia, postural middle cerebral artery
instability, mild ataxia, and postural hypotension. What is e) patients with transcortical type aphasias have a lesion
the likely diagnosis? which damages both Wernicke’s and Broca’s cortical
a) vitamin B12 deficiency areas
b) Parkinson’s disease
c) normal pressure hydrocephalus 12. A patient who is unable to use a comb but has no muscle
d) multiple systems atrophy weakness is likely to have a lesion in which part of the
central nervous system?
a) the hand area of the motor cortex
7. Patient with dizziness, left arm and leg weakness, loss of
b) the vermis of the cerebellum
pain and temperature senasation in the left face and
c) Broca’s area
right extremeties. What is the likely diagnosis? d) part of the premotor or prefrontal cortex
a) lateral medullary syndrome e) posterior parietal cortex
b) migraine with aura
c) middle cerebellar artery territory stroke
d) benign paroxysmal positional vertigo (BPPV)

MCCQE 2002 Review Notes Sample Questions – 25


NEUROSURGERY
1. Which of the following is FALSE regarding intracranial 9. A 70 year-old man complains of trouble walking. On
dynamics? examination, you find that he has left leg weakness and
a) normal intracranial pressure is 6-15 mmHg hyperreflexia, but relatively normal power and reflexes in
(8-18 cm H2O) his right leg. Furthermore he has decreased position and
b) the relationship between an expanding intracranial
mass and the resultant rise in intracranial pressure vibration sense in his left leg relative to his right, but
is linear decreased pin prick sensation in his right leg relative to
c) cerebral blood flow depends on cerebral perfusion his left. MRI of his lumbar spine reveals a tumour
pressure and cerebral vascular resistance compressing the man’s spinal cord. Where is the tumour?
d) lumbar puncture is contraindicated in patients with a) lateral to the right side of the lumbar spinal cord
known or suspected intracranial mass lesions b) lateral to the left side of the lumbar spinal cord
c) within the centre of the lumbar spinal cord
2. Which of the following is NOT a classic finding in d) anterior and central to the lumbar spinal cord
someone with raised intracranial pressure?
a) headache with nausea and vomiting
10. A 29 year-old bodybuilder presents complaining of right
b) respiratory changes
c) increased blood pressure arm pain and weakness. Examination reveals a
d) tachycardia decreased triceps reflex on the right and decreased
sensation on the right middle finger. MRI shows a right
3. An obsese 30 year-old woman presents with headache posterolateral intervertebral disc hernation. Where is the
and nausea, and a bilateral decrease in visual acuity. herniation ?
She has no recent history of trauma and is otherwise a) C4-5 interspace
well, but takes tetracycline for acne. CT and MRI scans of b) C5-6 interspace
her head with and without contrast are normal. Of the c) C6-7 interspace
following options, what is the most likely diagnosis? d) C7-T1 interspace
a) normal pressure hydrocephalus
b) benign intracranial hypertension
c) meningioma 11. What is the most common cause of an epidural
d) acute subdural hematoma hematoma?
a) ruptured middle cerebral atery
4. What is the most common type of primary brain tumour b) ruptured anterior cerebral artery
in adults? c) ruptured posterior communicating artery
a) astrocytoma d) ruptured middle meningeal arter
b) medulloblastoma
c) meningioma 12. What is the typical appearance of an acute subdural
d) vestibular schwannoma
hematoma on noncontrast CT?
5. Which of the following is NOT a characteristic CT feature a) diffuse intraparenchymal hypodense mass
of a brain abscess? b) hyperdense biconvex mass
a) perilesional hypodensity c) hypodense intraventricular mass
b) central hypodensity d) hyperdense concave mass
c) homogenous contrast enhancement
d) ring enhancement with contrast 13. Which of the following is NOT a classic finding in someone
with carpal tunnel syndrome?
6. A 45 year-old man presents with headache. On a) positive Tinel’s syndrome
examination, you notice that he has a large chin and b) hand pain sometimes awakening patient at night
brow, as well as large clammy hands. MRI of his head
c) hypothenar muscle wasting
reveals a large midline suprasellar mass. What visual
deficit might you expect in this gentleman? d) positive Phalen’s sign
a) right monocular blindness
b) amaurosis fugax 14. What is the most common location for pediatric brain
c) left homonymous hemiaopia tumours?
d) bitemporal hemianopia a) anterior cranial fossa
b) posterior cranial fossa
7. What is the most appropriat initial investigation in the c) middle cranial fossa
diagnosis of subarachnoid hemorrhage? d) sella turcica
a) lumbar puncture
b) CT without contrast
c) MRI 15. What is the term for the group of hindbrain abnormalities
d) CT with contrast involving some degree of cerebellar hernation or
hypoplasia?
8. Which of the following is NOT part of the management a) Chiari malformation
of vasopasm following subarachnoid hemorrhage? b) Dandy-Walker malformation
a) maintaining blood pressure at or below 120/80 c) craniosynostosis
b) nimodipine d) myelomeningocele
c) IV solutions to decrease hematocrit levels
d) angioplasty

26 – Sample Questions MCCQE 2002 Review Notes


OBSTETRICS
1. Polyhydramnios is associated with all of these EXCEPT: 9. Which of the following is TRUE regarding malpresentation
a) premature labour of a fetus?
b) cord prolapse a) It is associated with an increased risk of congenital
c) postpartum hemorrhage anomaly
d) Potter’s syndrome b) Compound presentation precludes a vaginal delivery
e) trisomy 18 c) It commonly occurs in association with an anthropoid
pelvis
2. With regard to thromboembolism and pregnancy all of d) It may be corrected during labour by intravenous
the following are true EXCEPT: oxytocic agents
a) Risk increases with maternal age e) It should be treated by immediate stabilizing
b) Warfarin is teratogenic induction if the membranes rupture
c) There is an increase in factors I, VII, VIII, IX, X, XII
d) Heparin crosses the placenta 10. All of the following are true about the renal system in
e) The uterus compresses veins contributing the pregnancy EXCEPT:
venous stasis a) The GFR is 60% greater than normal by 12 weeks
b) There is a link between UTI and low birth weight
3. Which of the following are TRUE with regard to normal c) Acute glomerulonephritis is a rare condition and is
labour: usually diagnosed as preeclampsia
a) Labour commences with onset of regular painful d) Asymptomatic bacteriuria has a frequency of 5%
contractions in the presence of a dilated cervix e) The creatinine and urea decrease in the normal
b) Retraction of the head during the second stage woman compared with non-pregnant values
suggests an undiagnosed malposition
c) The average time for the second stage is one hour in 11. The diagnosis of pregnancy is related to which of the
the multiparous woman following?
d) The second stage includes both a latent and active a) May be delayed as there is commonly a small
stage amount of blood loss at the time of the first missed
e) Signs of placental separation include a gush of bright period
red placental blood b) Nausea and vomiting usually precede amenorrhea
c) Portable doppler machine may detect a fetal heart at
4. Which of the following is associated with profuse painless 6 weeks
vaginal bleeding in the third trimester? d) In the absence of biophysical and biochemical
a) abruptio placenta diagnostic aids a bimanual examination to detect
b) umbilical cord prolapse Hegar’s sign should be performed
c) degenerating fibroid e) The beta subunit of HCG is similar to corresponding
d) placental insufficiency units on LH,FSH and TSH giving false positive results
e) placenta previa on immunosorbent assays
5. Which of the following is NOT a prerequisite to the use of 12. All of the following are prerequisites for labour
forceps in obstetrical delivery? suppression (tocolysis) EXCEPT:
a) patient must be fully dilated a) intact membranes
b) full bladder b) live fetus
c) adequate anaesthesia c) absence of fetal distress
d) position of fetus known d) no dilatation
e) ruptured membranes e) necessary personnel if tocolysis fails
6. The most common cause of postpartum hemorrhage is: 13. With regards to fetal heart monitoring, which of the
a) retained placenta following is TRUE:
b) uterine atony a) Early decelerations are related to a vagal response
c) vaginal lacerations to head compression
d) cervical lacerations b) An acceleration is defined as an increase of at least
e) uterine fibroids 20 bpm lasting at least 20 seconds
c) Normal range is 100-180 bpm
7. Which of the following is NOT associated with severe d) A variable deceleration is the least common change
preeclampsia? seen during labour
a) elevated liver transaminases e) A late deceleration peaks at the same time as the
b) thrombocytopenia uterine contraction
c) blood pressure greater than 160/110
d) oliguria 14. Which of the following is NOT true:
e) hematemesis a) With rubella infection, the greatest risk to the fetus
occurs in the first trimester
8. A 28 year old G1P0 woman at 35 weeks presents to the b) Genital herpes lesions is an indication for a caesarean
labour floor with painless vaginal bleeding. Which of the section
following should NOT be done? c) AZT decreases the incidence of vertical transmission
a) vaginal examination of HIV
b) complete blood count d) Varicella vaccine is safe during pregnancy
c) crossmatch blood e) All women should be screened for Hepatitis B
d) ultrasound
e) fetal heart rate monitor 15. Which of the following is TRUE:
a) The puerperium refers to the first 2 weeks after
delivery
b) The uterus should reach the non-pregnant state with
in 1-2 weeks
c) Foul smelling lochia suggests endometritis
d) Lochia changes in time from lochia rubra to lochia
serosa to lochia alba
e) Postpartum blues are rare

MCCQE 2002 Review Notes Sample Questions – 27


OPHTHALMOLOGY
1. Which of the following is NOT a contraindication to pupil 10. Severe photophobia is most characteristic of:
dilation: a) acute glaucoma
a) narrow anterior chamber b) cataracts
b) iritis c) iritis
c) neurologic abnormality requiring pupillary d) corneal abrasion
evaluation
d) iris supported anterior chamber lens implant 11. Which is NOT a management step in iritis:
a) pressure patch
2. In myopics, the eyeball is: b) pupil dilation
a) too long c) systemic analgesics
b) too short d) medical work-up to determine etiology
c) non-spherical
d) crooked 12. Cataracts can be due to all of the following but:
a) aging
3. The most common cause of exophthalmos in children is: b) diabetes mellitus
a) hyperthyroidism c) increased lipids
b) orbital cellulitis d) uveitis
c) orbital tumours
d) orbital hemorrhage 13. Which of the following require the most urgent attention:
a) central retinal vein occlusion
4. Eversion of the lower lid margins is termed: b) cataracts
a) trichiasis c) scleritis
b) entropion d) central retinal artery occlusion
c) ectropion
d) chalazion 14. Management for primary angle closure galucoma
includes all of the following except:
a) laser iridotomy
5. The most common type of lid carcinoma is: b) mydriatic drops
a) adenocarcinoma c) topical beta-blockers
b) squamous cell carcinoma d) IV hypertonic mannitol
c) sebaceous cell carcinoma e) systemic carbonic anhydrase inhibitors
d) basal cell carcinoma
15. Which of the following is NOT associated with primary
6. Which conjunctivitis typically begins as unilateral and angle closure glaucoma:
progresses to the opposite eye: a) steroid use
a) bacterial b) nausea and vomiting
b) viral c) hyperopia
c) allergic d) painful red eye
d) chlamydial e) opacified cornea

7. Which of the following is associated with contact lens 16. Which of the following is NOT a cause of a relative
wear: afferent pupillary defect:
a) vernal conjunctivitis a) multiple sclerosis
b) iritis b) optic neuritis
c) scleromalacia perforans c) dense cataract
d) giant papillary conjunctivitis d) large retinal detachment
e) central retinal vein occlusion
8. Which of the following should never be given for corneal
abrasions: 17. Which of the following is associated with impaired pupil
a) topical analgesics dilation:
b) topical antibiotics a) sympathetic stimulation
c) pressure patch b) Adie’s tonic pupil
d) topical cycloplegics c) parasympathetic understimulation
d) Horner’s syndrome
9. Dendritic lesions are characteristic of:
a) herpes zoster keratitis 18. The most common ocular infection in HIV is:
b) arcus senilus a) herples simplex
c) herpes simplex keratitis b) pnuemocystis carinni
d) scleritis c) cytomegalovirus
d) candida
e) toxoplasmosis

28 – Sample Questions MCCQE 2002 Review Notes


OPHTHALMOLOGY . . . CONT.

19. In a newly diagnosed patient with type 2 diabetes, 22. Which of the following tests for a phoria:
appropriate screening for retinopathy should involve: a) Hirschberg test
a) begin screening five years after diagnosis b) cover test
b) repeat in four years and thereafter annually c) cover-uncover test
c) begin screening three years after diagnosis, then d) holler test
repeat annually e) accomadation reflex
d) begin screening after age 50; earlier if poor glycemic
control 23. The following are true in the management of chemical
burns to the eye EXCEPT:
20. Non-proliferative changes in diabetic retinopathy include a) alkali burns have a worse prognosis than acid burns
all of the following EXCEPT: b) an alkali burn should be neutralized immediately
a) microaneurysms c) an IV drip with water should be set-up for irrigation in
b) retinal edema the emergency room
c) intraretinal microvascular anomalies d) cyclopegic drops should be administered
d) retinal edema
e) dot and blot hemorrhages 24. In differentiating the causes of red eye, photphobia is
most characteristic of:
21. Grade 5 involvement in graves disease refers to: a) cataract
a) corneal involvement b) conjunctivitis
b) soft tissue involvement c) angle closure keratitis
c) sight loss due to optic neuropathy d) acute glaucoma
d) proptosis e) acute iritis

MCCQE 2002 Review Notes Sample Questions – 29


ORTHOPEDICS
1. A 35 year old male manual labourer sustained a 4. A 21 year old presents to your office after injuring her
displaced subcapital hip fracture after falling at work. knee in a soccer game. She states that the knee clicks
The fracture was reduced and fixed with 3 cannulated when she walks and has “locked” on several occasions.
screws. Five months after the operation, he presents to On exam there is an effusion and the knee is grossly
your clinic with worsening hip pain. What is the most stable. The most likely diagnosis is:
likely diagnosis? a) Anterior cruciate ligament tear
a) Nonunion b) Meniscal tear
b) Osteonecrosis c) Osteoarthritis
c) Loosening of the cannulated screws d) Bursitis
d) Malunion e) Medial collateral ligament tear
e) Osteomyelitis
5. All of the following regarding Achilles tendon rupture are
2. A 25 year old professional basket ball player lands on his true, EXCEPT:
right foot while it is in a pronated and externally rotated a) Positive Thompson’s test
position. He hears a “snap” and is unable to weight bear b) Palpable gap over Achilles tendon
on his right foot. An ankle X-ray reveals significant talar c) Weak plantar flexion
shift without a visible fibular fracture. The most d) May occur secondary to steroid injection
appropriate next step is: e) Treat by casting foot in dorsiflexion
a) Closed reduction of the ankle mortise followed by
application of a cast
b) Bedrest for 4 weeks followed by intense
physiotherapy
c) X-ray the right knee to rule out a Maisonneuve
fracture
d) Open reduction with internal fixation of the right
ankle
e) Splint ankle and encourage weight bearing

3. A 75 year old lady slips on a throw rug in her living room


and falls. An X-ray reveals a displaced subcapital hip
fracture. Prior to her fall, the patient lived alone,
per formed all ADLs independently and enjoyed golfing.
The most appropriate management of this fracture is:
a) Moore’s unipolar hemiarthroplasty
b) Bedrest for 6 weeks
c) Reduction with internal fixation using 3 cannulated
screws
d) Bipolar hemiarthroplasty
e) Total hip replacement

30 – Sample Questions MCCQE 2002 Review Notes


OTOLARYNGOLOGY
1. Regarding laryngeal cancer, which of the following is 8. The diagnosis in question 7 may be confirmed by:
FALSE? a) a positive Wasserman test
a) hoarseness appears early b) a positive lupus erythematosus (LE) preparation
b) involved nodes are not palpable in 35% of cases c) a biopsy of the lesion
c) distant metastasis appears early d) diagnostic mandibular and maxillary x-rays
d) direct extension is common e) observation of further progression of the disease
e) it is 90% five-year curable when limited to one cord
9. A cholesteatoma is:
2. All of the following are removed in radical neck a) an atherosclerotic lesion
dissection EXCEPT: b) a dermal collection of cholesterol salts
a) sternocleidomastoid muscle c) epithelial debris in the middle ear
b) external carotid artery d) a yellow papule beneath the oral tongue
c) internal jugular vein e) retained cerumen
d) spinal accessory nerve
e) submaxillary gland 10. Small, malignant tumours of the larynx that are intrinsic in
origin and have not spread beyond the larynx are BEST
3. Which of the following factors is NOT associated with treated by:
squamous cell carcinoma of the larynx? a) irradiation
a) male sex b) laryngofissure
b) age in fifth and sixth decades c) total laryngectomy
c) history of woodworking d) total laryngectomy and radical neck dissection
d) large ethanol intake e) radium needle implants
e) tobacco smoking
11. Clinical features of facial fractures frequently include all of
4. In LeFort I fractures, the fragment consists of all of the the following EXCEPT:
following EXCEPT: a) deformity
a) upper teeth and palate b) facial nerve paralysis
b) lower portions of the pterygoid processes c) anesthesia over areas of trigeminal branch
c) portions of the walls of both maxillary antra distribution
d) nasal spine d) ocular disparity
e) bridge of the nose e) malocclusion of the teeth

5. In general, traumatic perforations of the tympanic 12. The MOST sensitive test for nasal fracture is:
membrane: a) history
a) are a surgical emergency b) physical diagnosis
b) will heal spontaneously in most cases c) plain x-ray studies
c) usually require operative repair d) magnetic resonance imaging
d) require microsurgical repair e) computed tomography (CT) scanning
e) require a graft for repair
13. Mixed tumours of the salivary gland:
6. The MOST common organism in acute otitis media of a) are most common in the submaxillary gland
older children and adults is: b) are usually malignant
a) Staphylococcus c) are most common in the parotid gland
b) Streptococcus d) usually cause facial paralysis
c) Hemophilus influenzae e) are associated with calculi
d) Klebsiella pneumoniae
e) Pseudomonas 14. In epistaxis, what percentage of the cases will respond to
ten minutes of direct pressure?
7. A 65-year-old white male who has been smoking pipes a) 10%
since early adulthood notes a small patch of white on the b) 30%
lateral anterior portion of the tongue. The patch is not c) 70%
painful for the first month, but gradually becomes more d) 90%
painful as it begins to enlarge and ulcerate. The MOST e) 0%
likely diagnosis is?
a) benign nonspecific ulceration 15. Which of the following is NOT a cause for conductive
b) leukoplakia (benign) hearing loss?
c) epulis a) otitis media
d) carcinoma of the tongue b) otosclerosis
e) ranula of the tongue c) noise-induced hearing loss
d) perforation of the tympanic membrane
e) ossicular chain disruption

MCCQE 2002 Review Notes Sample Questions – 31


OTOLARYNGOLOGY . . . CONT.

16. Conductive hearing losses are usually reversible. Which 23. What is the MOST common cause of acquired subglottic
of the following conditions is reversible by surgical stenosis?
treatment? a) motor vehicle trauma
a) otosclerosis b) prolonged endotracheal intubation
b) presbycusis c) chronic bronchitis
c) sudden hearing loss d) tracheoesophageal fistula
d) ototoxicity e) previous tracheal surgery
e) meningitis
24. A 5-year-old child has persistent serous effusions in both
17. What is the BEST treatment for most cases of ears for 6 months after a routine acute infection. He has a
sensorineural hearing loss associated with 40-dB condutive heraring loss in both ears and has been
aging (presbycusis)? having trouble in school. What would be the BEST
a) nothing treatment for this child?
b) hearing aid a) observe the child for another 3 months
c) ear trumpet b) prescribe amoxicillin for 10 days
d) diuretic therapy c) recommend hearing aids
e) labyrinthectomy d) place ventilating tubes
e) prescribe prophylactic antibiotics for 3 months
18. The MOST common benign lesion of the external
ear is: 25. A 3-year-old child has has eight episodes of acute otitis
a) melanoma media in 6 months and has difficulty resolving the
b) chondrodermatitis nodularis chronicus helicus effusions between infections. What should be done to
c) cerumenoma effectively eliminate the infections?
d) actinic keratosis a) continuing treating each infection as it arises
e) exostosis of the canal b) place ventilating tubes
c) prescribe prophylactic antibiotics for 6 months
19. MOST of the infectious and/or inflammatory diseases d) remove the tonsils
involving the middle ear space are secondary to: e) give IV antibiotics for 4 weeks after infectious
a) ciliary dyskinesia disease consultation
b) resistant pathogens
c) eutstachian tube dysfunction 26. The following clinical entities are common causes for
d) tobacco abuse tinnitus EXCEPT:
e) allergic diathesis a) high-frequency hearing loss
b) Ménière’s disease
20. Acute otitis is: c) ototoxic drugs
a) a rare condition d) loud noise exposure
b) the most common reason ill children visit the doctor e) acute otitis media
c) usually not accompanied by pain and fever
d) caused by coliform bacteria 27. Vertigo is very common in all of the following conditions
e) treated by placing ventilating tubes EXCEPT:
a) vestibular neuritis
21. All of the following ar indications for tonsillectomy
b) Ménière’s disease
EXCEPT:
c) presbycusis
a) six to seven episodes of tonsillitis in 1 year
d) viral labyrinthitis
b) airway obstruction secondary to tonsillar
e) benign paroxysmal positional vertigo
hypertrophy
c) repeat ear and sinus infections
28. The fastest, safest means of establishing a surgical
d) recurrent peritonsillar abscess
airway is:
e) very large asymmetric tonsil in an adult
a) endoscopic intubation
22. The MOST common cause for infant stridor, accounting b) tracheotomy under local anesthesia
or 60% of the cases, is: c) tracheotomy under general anesthesia
a) subglottic hemangioma d) cricothyrotomy
b) vocal cord paralysis e) puncture through the thyroid membrane
c) laryngomalacia
d) congenital webs
e) laryngeal cleft

32 – Sample Questions MCCQE 2002 Review Notes


PEDIATRICS
1. A 6 month-old infant presents in the winter with fever, cough, 6. Which of the following investigations is most helpful in the
wheezing, tachypnea and decreased appetite. A chest assessment of a child presenting with an acute asthma attack, who
radiograph shows hyperaeration and streaky perihilar infiltrates responds poorly to treatment?
bilaterally. You diagnose bronchopneumonia. Which organism a) white cell count and differential
would most likely be causing this child's infection? b) arterial blood gases
a) Chlamydia pneumoniae c) chest x-ray
b) Mycoplasma pneumoniae d) pulmonary function tests
c) Streptococcus pneumoniae e) sweat chloride test
d) Haemophilus influenzae
e) respiratory syncytial virus 7. A 6 year-old boy is brought to emergency with a 5 day history of
fever, cough, and poor appetite, but no vomiting. On exam, he
The following case pertains to questions 2 and 3: appears unwell, is febrile, and has crusty nasal discharge and a
A 12 month-old girl is brought to the emergency department for the wet-sounding cough. Chest x-ray reveals a pulmonary infiltrate in the
second time in 2 days for vomiting and passage of 8 to 10 watery stools right middle lobe. He is admitted to hospital for pneumonia. What is
per day. the best management for this child?
a) observe for 24 hours with IV fluids only
2. Of the following, which provides the best estimate of the b) observe for 24 hours with IV fluids and acetaminophen
patient’s volume deficit: c) give oral amoxicillin and acetaminophen
a) weight change since the beginning of the illness d) give IV ampicillin and oral acetaminophen
b) hydration of mucous membranes, skin turgor, and e) give oral erythromycin and acetaminophen
level of consciousness
c) pulse, blood pressure, and peripheral capillary filling time 8. In comparing breast milk and formula, which of the following
d) serum electrolytes statements is NOT correct:
e) serum urea nitrogen and creatinine levels a) breast milk has a higher percentage of protein
b) breast milk has whey:casein ratio of 60:40
3. The patient has lost 0.6 kg. She is moderately lethargic and has c) breast milk contains leukocytes, complement and lysozymes
dry mucous membranes and reduced skin turgor. Blood pressure is d) breast milk has a lower concentration of iron than iron
80/40 mm Hg, and pulse is 120 per minute; capillary refill is fortified formulae
reasonably brisk. Lab studies reveal: sodium 131, potassium 4.8, e) breast milk has an optimal calcium:phosphorus ratio of 2:1
chloride 101, bicarbonate 16 mEq/L, urea nitrogen 24 mg/dL and
creatinine 0.6 mg/dL. The best strategy for managing this child 9. Most umbilical hernias in children:
is to: a) need strapping
a) hospitalize for administration of IV fluid therapy b) resolve spontaneously
b) administer an oral rehydrating solution while the c) require elective surgery
child is under medical supervision for 4-6 hours d) require immediate surgery
c) instruct the parents about oral rehydration at home e) are associated with a higher incidence of inguinal hernias
d) hospitalize after giving 20 mL/kg of 0.9% saline IV
e) instruct parents on use of soy formula 10. A newborn male spits up his first feeding and develops bilious
emesis with subsequent feedings. On physical exam he appears ill,
4. Which of the following cases is most suggestive of child abuse? has a scaphoid abdomen and absent bowel sounds. Abdominal x-ray
a) a child who clings to her parent but shies away from the shows air in the proximal small bowel, but a paucity of air in the
emergency physician. distal digestive tract. The most likely cause for this infant's vomiting
b) a parent who refuses to leave his/her child alone with the and clinical finding is:
physician, despite the physician’s repeated requests. a) antral web
c) a parent who claims that his child broke her arm after falling off b) choledochal cyst
her bike. c) Hirschsprung disease
d) a child with recurrent urinary tract infections despite antibiotic d) tracheoesophageal fistula
prophylaxis. e) volvulus
e) a child with old-looking bruises on both elbows and shins.
11. Breastfeeding is contraindicated if the mother has:
5) A 14 year-old girl has refused to go to school 3 times in the past a) acute EBV infection
4 months. She says, "My tummy hurts," but she cannot point to b) acute HepA infection
where it bothers her. Her appetite is good, her bowel movements c) chronic HepB infection
are normal, and she is sleeping well at night. She is a d) asymptomatic HIV infection
healthy-looking girl with no abnormal findings on physical exam. e) none of the above
What is the most likely diagnosis?
a) benign abdominal mass compressing her duodenum
b) diaphragmatic hernia
c) functional abdominal pain
d) gastroesophageal reflux disease
e) early peptic ulcer disease

MCCQE 2002 Review Notes Sample Questions – 33


PEDIATRICS . . . CONT.

12. A 10 month-old child develops low-grade fever and sunburn-like 19. Which of the following statements is true?
erythema over much of the body, but most prominently a) cancer is the second most common cause of death in children
in the intertriginous areas. Within 36 hours, sheet-like b) Hodgkin's lymphoma is the most common childhood cancer
desquamation is noted in the flexures and around the mouth. c) Hyperploidy in leukemic cells is a poor prognostic indicator
The mucous membranes are spared. Which of the following is d) Wilm's tumor is rarely associated with other congenital
the most likely diagnosis: abnormalities
a) Kawasaki disease e) neuroblastomas usually occur in late adolescence
b) staphylococcal scalded skin syndrome
c) Stevens-Johnson syndrome 20. A 5 year-old girl with hypogammaglobulinemia and absent
d) toxic epidermal necrolysis immunoglobulin A (IgA) receives infusions of immune
e) toxic shock syndrome globulin monthly. Shortly after her most recent infusion
began, she developed hypotension, wheezing, and several
13. Which of the following is a contraindication to breast feeding urticarial lesions. Which of the following best explains her
in Canada: reaction?
a) allergic disease in the family a) anaphylactic reaction due to IgE anti-IgA antibodies
b) infantile diarrhea b) gram-negative sepsis due to contaminated immune
c) marijuana smoking globulin
d) HIV infection c) idiosyncratic reaction due to rapid infusion of immune
e) maternal use of ibuprofen globulin
d) serum sickness reaction from foreign serum in the immune
14. Central cyanosis in the newborn infant is most often caused by: globulin
a) congenital heart disease e) transfusion reaction due to ABO incompatibility
b) lung disease
c) central nervous system disease 21. When prophylactic antibiotic therapy is used for tooth
d) methemoglobinemia extraction in a patient with a ventricular septal defect:
e) hypoglycemia a) therapy is started 24 hours prior to the procedure
b) a throat swab should be taken prior to the procedure
15. In a premature infant who is suspected of having necrotizing c) a second generation cephalosporin (cefuroxime) is the
enterocolitis (NEC), each of the following is correct EXCEPT: therapy of choice
a) septicemia is associated with an increased risk of NEC d) therapy is given for one week following the extraction
b) NEC is thought to be caused by systemic hypertension e) erythromycin is the drug of choice for those allergic to penicillin
c) the finding of air in the portal vein indicates severe illness
d) respiratory distress increases the risk of NEC 22. Which of the following statements about stuttering in a
e) Apgar scores inversely correlate with the risk of NEC 4 year-old is incorrect:
a) it is characterized by intermittent difficulty in producing a
16. Which of the following is the most significant risk factor for the smooth flow of speech
development of childhood asthma: b) it is more than 3 times more frequent in girls than in boys
a) family social background c) it is exacerbated by anxiety
b) parental asthma d) more than 30% of children who stutter recover spontaneously
c) stress in the family e) none of the above
d) parental smoking
e) presence of pets in the house 23. A woman who is positive for hepatitis B surface antigen
(HBsAg), but negative for hepatitis B antigen (HBeAg),
17. Which of the following statements about sickle cell disease is true? delivers at term. What would be the best management for
a) all patients with sickle cell disease have a homozygous HbSS this woman's infant?
genotype a) administer gamma globulin intramuscularly immediately
b) sickle cell disease causes a severe chronic anemia that is not and at 1 month of age
routinely transfusion dependent b) administer hepatitis B (HB) vaccine immediately and at 1
c) patients have increased susceptibility to infection by month and 6 months of age
nonencapsulated organisms c) administer hepatitis B immune globulin (HBIG) if cord
d) patients usually present with sickle cell crises within one blood is positive for HBsAg
month of age d) administer HBIG and HB vaccine immediately, and HB
e) splenic dysfunction usually does not occur until the child vaccine again at 1 month and 6 months of age
enters his/her teens e) advise mother that breastfeeding is contraindicated

24. Regarding sexual abuse of children, each of the following


18. Which of the is more characteristic of platelet abnormalities and
statements is true EXCEPT:
not coagulation defects? a) no genital injury is found in the majority of patients
a) hematomas b) father-daughter incest is more common than brother-sister
b) hemarthrosis incest
c) petechiae c) most assailants are unknown to the victim
d) minimal bleeding from small cuts d) laboratory findings usually show no presence of sperm
e) positive family history e) half of the abused children come from single parent families

34 – Sample Questions MCCQE 2002 Review Notes


PEDIATRICS . . . CONT.

25. The viral infection most likely to cause CNS involvement and focal 32. A 16 year-old male has a 4 day illness of abrupt onset consisting of
neurological findings is : fever, sore throat, rhinorrhea, cough and mild abdominal pain. He
a) coxsackievirus has a red tonsillopharyngeal area, a whitish exudate on the tonsils
b) herpes simplex and tender anterior cervical nodes. The most likely cause of his
c) enterovirus infection is:
d) rabies a) adenovirus
e) rhinovirus b) Group A beta-hemolytic Streptococcus
c) Epstein-Barr Virus
26. Which of the following predisposes infants to chronic otitis media? d) Neisseria gonorrhea
a) bottle-feeding in upright position e) Staphylococcus
b) abnormal tympanic membrane formation
c) environmental factors such as daycare and passive smoking 33. A 13 month-old infant boy has chronic diarrhea, poor appetite,
d) allergies irritability and growth failure. He had been well previously,
e) none of the above developing until diarrhea began at 3 months of age. Findings
include weight loss less than 5th percentile and length at 25th
27. All infants less than 3 months of age who have fever and no percentile, cachectic appearance, wasted extremities and
localizing signs should have all of the following evaluations protuberant abdomen. His labs are: albumin 23, protein 40. Stool
EXCEPT: positive for reducing sugars and negative for enteric pathogens
a) blood cultures and ova and parasites. These findings are most consistent with
b) careful history and physical exam a) Celiac disease
c) chest x-ray b) cow milk allergy
d) CBC (with differential) c) Crohn’s disease
e) urine culture d) cystic fibrosis
e) none of the above
28. All the following statements regarding transient tachypnea of the
newborn (TTN) is true EXCEPT: 34. Which of the following is most likely to provide the basis for
a) infants born by C-section are at increased risk for making the correct diagnosis in an infant or child who has failure to
developing TTN thrive (FTT)?
b) residual pulmonary function disability is common among a) blood chemistries
infants who have TTN b) cultures
c) the incidence of TTN is higher than Respiratory Distress c) history and physical
Syndrome (RDS) among term infants d) radiograph studies
d) TTN shows marked improvement with 12-24 hrs e) none of the above

29. Antibiotic prophylaxis against infective endocarditis is required for 35. A 8 year-old boy has had paroxysmal abdominal pain since his
all of the following EXCEPT: parents separated 6 months ago. Which of the following symptoms
a) rheumatic valve lesions would support an organic basis for his disease?
b) prosthetic heart valves a) headaches accompanies the pain
c) isolated secundum atrial septal defect b) pain located in the periumbilical region
d) pacemaker leads c) pain awakens child at night
e) patent ductus arteriosus d) symptoms last < 1hr
e) none of the above
30. Causes of microcytic anemia include all of the following EXCEPT:
a) excessive cow's milk intake 36. The most common cause of chronic diarrhea in a
b) iron deficiency 6 month - 36 month old child is:
c) folic acid deficiency a) chronic non-specific diarrhea
d) thalassemias b) disaccharidase deficiency
e) lead poisoning c) enteric infection
d) malabsorption
31. The latest time after the onset of group A ß hemolytic e) protein intolerance
Streptococcus pharyngitis that initiation of penicillin therapy can be
expected to prevent acute rheumatic fever is: 37. Which of the following is NOT required for a diagnosis of juvenile
a) 24 hrs rheumatoid arthritis?
b) 48 hrs a) arthritis in at least one joint
c) 96 hrs b) arthritis lasting for at least 6 weeks
d) 9 days c) positive rheumatoid factor
e) 2 weeks d) onset before the age of 16
e) other causes of arthritis excluded

MCCQE 2002 Review Notes Sample Questions – 35


PEDIATRICS . . . CONT.

38. A child presents with bilateral shin pain. Which of the following 45. All of the following are appropriate in the initial assessment of a
suggests that this is NOT growing pains? child with failure to thrive EXCEPT:
a) pain is poorly localized a) diet history
b) pain awakens the child at night b) social history
c) no fever or rash c) measurement of height, weight and head circumference
d) pain abates with reassurance and massage d) growth hormone levels
e) child may limp in the morning from stiffness e) bone age x-ray

39. Which of the following steps is NOT indicated in the management 46. Which of the following statements about a child with short stature
of croup? is CORRECT?
a) keeping the child calm a) the bone age is delayed in a child with constitutional growth
b) hydration delay
c) antipyretics b) height crosses a major percentile line on the child’s growth
d) antibiotics chart
e) humidified oxygen c) weight is more affected than height in a child with an
endocrine deficiency
40. Which of the following pairs shows the CORRECT stage of normal d) karyotyping part of the routine investigation of all children with
development? short stature
a) 6 months - pulls to stand e) growth hormone replacement is helpful in a child with familial
b) 12 months - pincer grasp growth delay
c) 18 months - handedness
d) 2 years - draws a cross 47. Which of the following developmental milestones in not a cause for
e) 3 years - tells a story concern?
a) 15 month old not walking
41. Which of the following is NOT characteristic of a functional b) 12 month old not talking
murmur? c) 12 month old who does not search for hidden objects
a) pansystolic murmur d) 3 year old unable to stand on one foot momentarily
b) murmur varies with position e) 6 month old with a persistent grasp reflex
c) variably split S2
d) murmur becomes louder with fever 48. All of the following immunizations should be administered to a
e) no extra clicks 7 year-old child who is a recent immigrant, with unknown
vaccination status EXCEPT:
42. Which of the following is the most common congenital heart lesion a) tetanus
in children? b) MMR
a) atrial septal defect c) diptheria
b) ventricular septal defect d) pertussis
c) patent ductus arteriosus e) polio
d) Tetralogy of Fallot
e) coarctation of the aorta 49. A 3 year old girl has a 2 month history of left knee swelling and
morning stiffness. There is no history of fever or rash. The child
43. Choose the INCORRECT statement about breastfeeding: appears very healthy. The most likely tentative diagnosis is:
a) for healthy term babies, breastfeeding is recommended over a) pauciarticular juvenile rheumatoid arthritis (JRA)
formula feeding b) systemic lupus erythematosus (SLE)
b) breastmilk contains more amino acids than cow’s milk. c) rheumatic fever
c) exclusively breastfed babies should be supplemented with d) Kawasaki’s disease
iron after 6 months of age e) Henoch-Schonlein purpura
d) assessment of adequate intake can be done by counting
the number of wet diapers in a day 50. An 8 year old boy has had 2 episodes of loss of awareness. These
e) an HIV-positive mother should not breastfeed last 5 seconds and consist of eyes and head turning to one side.
EEG reveals 3Hz spike and wave activity. First line therapy would
44. All of the following are characteristics of a child with Pervasive include:
Developmental Disorder (PDD) EXCEPT: a) ACTH
a) abnormal speech pattern b) phenytoin
b) stereotypic behaviours, such as head-banging or hand-flapping c) carbamazepine
c) abnormal cognitive function d) ethosuximide
d) consuming interest in one topic or activity e) phenobarbital
e) tendency to reciprocate in peer interactions

36 – Sample Questions MCCQE 2002 Review Notes


PEDIATRICS . . . CONT.

51. Turner syndrome is associated with the following clinical features 55. A 1100 gram infant born at 36 weeks gestation develops respiratory
EXCEPT: distress syndrom and requires supplemental oxygen until 4 months
a) major learning difficulties of age. A CXR then shows hyperinflation of both lungs and cystic
b) short stature changes in the bases. The most likely diagnosis is:
c) primary amenorrhea a) bronchopulmonary dysplasia
d) broad chest b) cor pulmonale
e) wide carrying angle at elbows c) presistant pulmonary hypertension
d) pneumonia
52. Which statement concerning delayed puberty is false? e) cystic adenomatoid malformation of the lung
a) defined absence of pubertal development by age 13 in girls
and age 14 in boys 56. Breat feeding is a contraindication to which of the following
b) the most common cause is constitutional delay vaccinations?
c) chronic disease may cause growth delay a) diptheria
d) delayed puberty in males is more suggestive of pathology than b) influenza
delayed puberty in females c) MMR
e) bone age is an important investigation in determining the d) hepatitis B
cause of delayed puberty in both males and females e) none of the above

53. A 3 year old Asian girl presents with a fever of 6 days, red lips and 57. A 6 month old infant presents in the winter with fever, cough,
tongue, bilateral nonpurulent conjunctivitis, cervical wheezing, tachypnea and decreased appetite. A CXR shows
lymphadenopathy and a polymorphic rash. hyperaeration and streaky perihilar infiltrates bilaterally. You diag
The most likely diagnosis is: nose bronchopneumonia. Which organism whould likely be causing
a) Streptococcal pharyngitis this child’s infection?
b) Kawasaki’s disease a) chlamydia pneumonia
c) infectious mononucleosis b) mycoplasma pneumonia
d) adenoviral infection c) streptococcus pneumonia
e) none of the above d) haemophilus influenza
e) respiratory syncitial virus
54. Which of the followng statements concerning minimal lesion
glomerulonephritis is correct? 58. Prostaglandin E1 infusion can be used for the initial management in
a) it is a common cause of nephrotic syndrome in children, but it each of the following neonatal heart lesions EXCEPT:
is a very rare cause of nephrotic syndrome in adulthood a) pulmonary atresia
b) glomerular filtration rate is usually preserved b) coarctation of the aorta
c) it is usually unresponsive to treatment such as prednisone and c) Tetralogy of Fallot
cyclophosphamide d) patent ductus arteriosus
d) it is most often accompanied by severe hypertension e) hypoplastic left heart syndrome
e) the urine typically shows red blood cell casts (suggestive of
proliferative glomerulonephritis)

MCCQE 2002 Review Notes Sample Questions – 37


PHARMACOLOGY
1. The first pass effect refers to: 9. A drug with a half life of 24 hours will take ____ to reach
a) the first time the drug reaches its site of action steady state:
b) passage of the drug from the skin into the body a) 12 hours
c) metabolism of the drug by the liver b) 24 hours
d) passage of the drug from the lung into the systemic c) 48 hours
circulation d) 96 hours
e) metabolism of the drug by kidneys e) 120 hours

2. A patient with hypoalbuminemia requires: 10. Regarding the figure below


a) a higher dose of a highly protein bound drug
b) the same dose of drug protein binding doesn’t A
100% C
matter

Response e.g. % of
c) a lower dose of a highly protein bound drug

receptors blocked
d) no drug at all its just too dangerous 60% B

3. Compared to water soluble drugs lipid soluble drugs: 50%


a) are larger molecules Potency
25% A>B>C
b) have higher ionizations
Efficacy
c) diffuse more easily through membranes A=C>B
d) more easily absorbed if given IV

4. The primary site of absorption of most drugs is the: Log Dose


a) mouth
b) skin
c) stomach
d) small intestine a) Drug A is less potent than drug B
e) lungs b) Drug A is less efficacious than drug B
c) Drug A is more efficacious than drug C
5. The primary site of biotransformation of most drugs d) Drug A is equally efficacious to drug C
is the: e) Drug A is equally potent to drug C
a) liver
b) lung 11. A drug with a narrow therapeutic index:
c) small intestine a) never requires therapeutic drug monitoring
d) skin b) always require therapeutic drug monitoring
e) kidneys c) can be given over a wide dose range with little worry
about toxicity
6. The main route of excretion for most drugs is the: d) has a TD50 at a lower concentration than the ED50
a) liver e) has an ED50 that is very close to the TD50
b) lung
c) small intestine 12. The following is TRUE regarding type A adverse drug
d) skin reactions
e) kidneys a) they account for 80% of all adverse drug reactions
b) they are an extension of the drugs pharmacological
7. Regarding phase I drug reactions which is NOT correct: effect
a) they are oxidation, reduction or hydrolysis reactions c) they are predictable
b) they involve conjugation with polar endogenous d) they require discontinuation of the drug
substrates e) they require a dose reduction of the drug
c) they increase the water solubility of the drug
d) they include the P450 mediated reactions

8. Regarding first order kinetics for drug metabolism which


is NOT true:
a) a constant fraction of drug is eliminated per unit
of time
b) elimination is based on drug concentration
c) drugs may go from zero to first order kinetics when
the receptor is saturated
d) it is a non-linear relation

38 – Sample Questions MCCQE 2002 Review Notes


PLASTIC SURGERY
1. A 23 year-old male was involved in a snowmobile 7. All of the following burns require extra fluid
accident and sustained a closed displaced humeral shaft administration EXCEPT:
fracture. His neurovascular status is normal. You realign a) burns greater than 70% TBSA
the fracture and place him in a splint. Prior to discharge b) electrical burn
he states he has numbness over the dorsum of this hand c) inhalation injury
and he is unable to extend his metacarpophalangeal d) pediatric burns
joints. Which of the following would you suspect to be e) 4º burns
the cause?
a) ulnar nerve was damaged at the time of the fracture 8. All of the following are complications of breast implants
b) radial nerve was damaged at the time of the fracture EXCEPT:
c) radial nerve is trapped in the fracture site a) capsular contraction
d) ulnar nerve is compressed by hematoma b) increased risk of infection
e) median nerve is trapped in fracture site c) breast asymmetry
d) systemic inflammatory reaction
2. A 55 year old female complains of numbness and
discomfort in her right hand, which is awakening her from 9. All of the following statements are true concerning a
sleep. Similar symptoms occur while steering the car hypertrophic scar EXCEPT:
or writing. You suspect carpal tunnel syndrome. a) it frequently improves with corrective surgery
Which of the following would lead you to consider and/or pressure
alternative diagnoses? b) it is more common in blacks
a) absence of sensory loss on testing with pinprick c) it does not outgrow its border
b) absence of thenar muscle wasting d) it occurs soon after surgery and usually
c) absence of weakness of abductor pollicis brevis subsides with time
d) presence of rheumatoid arthritic changes in the
hands 10. Regarding fractures of the mandible, all of the following
e) wasting of the thenar, hypothenar and interosseous are correct, EXCEPT:
muscles of the right hand a) they frequently are compound
b) they produce numbness of the lip
3. Which antibiotic is the first line drug of choice for an c) deformity and malocclusion are the usual
otherwise uncomplicated human bite infection of the presenting features
hand? d) the muscles of mastication keep fragments
a) Gentamicin from displacement
b) Penicillin e) the majority can be treated by dental fixation
c) Flagyl
d) Cephalexin 11. All of the following are true statements regarding wound
e) Erythromycin contractures EXCEPT:
a) contracture is due to dehydration of coagulum and is
4. Which of the following is correct regarding not an active, energy-dependent process
hemangiomas? b) contracture begins at about the fourth or fifth post
a) they usually will require surgery wound day
b) most present in the teenage years c) myofibroblasts move surrounding dermis inward
c) they usually resolve on their own d) contraction may be inhibited by cytotoxic drugs
d) most are found on the extremities e) contracture occurs in an inflammatory milieu
e) they are composed mostly of dilated veins
12. During wound healing, collagen is laid down by:
5. All the following are common conditions seen with a a) the endothelial cell
rheumatoid hand EXCEPT: b) the fibroblast
a) synovitis c) the wandering macrophage
b) radial drift d) the surrounding epithelium
c) thumb deformity e) none of the above
d) Swan neck deformity
e) Boutonniere deformity

6. A 23 year old man is involved in a house fire, sustaining


second and third degree burns to his face, anterior chest
and trunk, entire left arm, groin, and both legs
circumferentially. What is the estimated TBSA involved in
the burn?
a) 68.5%
b) 67.5%
c) 46%
d) 55%

MCCQE 2002 Review Notes Sample Questions – 39


PSYCHIATRY
1. Monoamine oxidase inhibitor drugs are used in the 8. All of the following are classified as paraphilias EXCEPT:
treatment of depression because they increase synaptic a) fetishism
levels of: b) homosexuality
a) gamma-aminobutyric acid (GABA) c) exhibitionism
d) sexual sadism
b) histamine
e) transvestism
c) acetylcholine
d) norepinephrine 9. A 32 year old engineer has been uncharacteristically
e) somatostatin active for several weeks. He spends most of his time at
work and gets little sleep. He has told another engineer
2. Neuropsychological effects of hallucinogens may include that he is involved “in a research project that will earn
all of the following EXCEPT: me the Nobel Prize”. Expensive research equipment
a) miosis keeps arriving at his office. The engineer is irritable, and
it is hard to hold his attention. A classmate from
b) tremor
graduate school recalls that the patient behaved in a
c) hyper-reflexia similar twice during stressful periods at school.
d) incoordination Long-term drug therapy for this patient would likely
e) blurred vision include:
a) haloperidol
3. Cocaine withdrawal can include all of the following b) valproic acid
EXCEPT: c) clozapine
a) “Crash” sleep d) ascorbic acid
e) chlordiazepoxide
b) anergia
c) anhedonia 10. Elderly male depressives typically present with all of
d) euphoria the following EXCEPT:
e) continued craving a) importuning
b) anxiety
4. Alcohol withdrawal includes all of the following EXCEPT: c) weight loss
a) autonomic hyperactivity d) little suicide risk
b) tremor e) insomnia
c) starts within 2-4 hours after prolonged drinking
11. From among the drugs listed below, which would be the
d) nausea cause for most concern in an overdose:
e) irritability a) paroxetine (SSRI)
b) amitriptyline (tricyclic)
5. Which would not be considered a risk factor for suicide in c) diazepam (benzodiazepine)
patients presenting with suicidal ideation: d) chlorpromazine (phenothiazine)
a) substance abuse e) fluoxetine (SSRI)
b) male gender
12. Which of the follow statements about schizophrenia
c) lack of social supports
d) unsuccessful attempt at suicide in the past is false?
e) childless marriage a) male schizophrenics to experience their first
psychotic episode at a younger age than women
6. A 54 year-old man has become forgetful, preoccupied, b) male schizophrenics are more frequently
withdrawn, irritable and disheveled. His physical hospitalized than female schizophrenics
examination was normal. The patient had been with his c) compared to young female schizophrenics, young
company for twenty-two years and was considered an male schizophrenics are at increased risk of
excellent employee. Which of the following is the most movement disorders secondary to neuroleptics
likely diagnosis: d) in women, the symptoms of schizophrenia tend to
a) multi-infarct dementia worsen after menopause
b) hypothyroidism e) all of the above statements are true
c) schizophrenia
d) alcoholism 13. A 29 year old school teacher who lives alone is brought to
e) major depression the emergency room because she has become
increasingly suspicious, hyperactive, and anorexic over the
7. Which of the following is correct about depression in
children: past two days. She believes that “people in the
a) family therapy should be avoided because it neighbourhood are out to get me”. She has not slept
scape goats a child who is already vulnerable in 2 nights. She reports seeing snakes crawling on the wall.
b) symptoms may manifest as antisocial behaviour Based on this information, the most likely diagnosis of the
c) antidepressants generally are not effective in woman’s problem is:
children a) anorexia nervosa
d) the suicide rate in children aged 8-13 is higher than b) cocaine withdrawal
it is in older adolescents c) paranoid personality
e) depression in children has been shown to be a d) psychostimulant abuse
prodrome to the later development of e) shared paranoid disorder
schizophrenia

40 – Sample Questions MCCQE 2002 Review Notes


PSYCHIATRY . . . CONT.

14. Anti-α-1-adrenergic blockade causes: 20. Which of the following statements concerning anorexia
a) nausea nervosa and bulimia nervosa is false:
b) constipation a) patients with either of these eating disorders are
c) orthostatic hypotension preoccupied with weight, food, and body shape
b) bulimia nervosa is more prevalent than is anorexia
d) dry mouth nervosa
e) drowsiness c) both of these eating disorders are more common in
females than in males
15. The following are common side effects of SSRIs EXCEPT: d) bulimia nervosa often presents earlier in
a) headache adolescence than does anorexia nervosa
b) sexual dysfunction e) bulimic symptoms may occur in both bulimia
c) vomiting nervosa and anorexia nervosa
d) anorexia
21. A pattern of unstable but intense interpersonal
e) orthostatic hypotension relationships, impulsivity, inappropriately intense anger,
identity disturbance, affective instability, and problems
16. Clozapine is the neuroleptic of choice for with being alone suggest a diagnosis of:
schizophrenia when: a) antisocial personality disorder
a) the patient shows no evidence of tardive dyskinesia b) narcissistic personality disorder
b) the patient has not improved with adequate c) histrionic personality disorder
dosages for 6 weeks of 3 other antipsychotics d) schizoid personality disorder
c) the patient is under fifteen years of age e) borderline personality disorder
d) the patient has a WBC count of >10 000 22. The criteria for diagnosis of a factitious disorder include:
e) the patient is very sensitive to anticholinergic a) intentional production or feigning of physical signs or
side effects symptoms
b) absence of secondary gain
17. Frequent conditions appearing comorbidly with ADHD c) possibility of economic gain
include: d) (a) and (b)
1) oppositional defiant and conduct disorders e) (a) and (c)
2) anxiety disorder
23. A patient with a fear of heights is brought to the top of a
3) learning disabilities tall building and required to remain there as long as
4) tic disorders necessary for the anxiety to dissipate. This is an
5) language disorders example of:
a) graded exposure
Which are correct: b) participant modeling
a) only 1 is correct c) positive reinforcement
b) 1, 2, and 3 are correct d) flooding
c) 2 and 4 are correct e) relationship therapy
d) all are true 24. Hypnosis has been used successfully in all of the following
conditions EXCEPT:
18. A 35 year old man presents to the emergency room with a) pain
suicidal ideation. He describes significant stress due to b) phobia
recent job loss and financial hardship. Further inquiry c) paranoia
reveals a history of repeated job loss, fraud charges, and d) anxiety
frequent arm slashing to decrease stress. The man was e) smoking
not disruptive as a child. The most likely diagnosis is:
25. Cognitive therapy helps to correct which of the following
a) borderline personality disorder cognitive distortions:
b) antisocial personality disorder a) depersonalization
c) adjustment disorder b) psychotic thinking
d) dysthymic disorder c) over-generalizations
e) schizophrenia d) selective inference
e) hallucinations
19. Which of the following statements concerning childhood
abuse and neglect is false: 26. The most frequently reported side effect of tricyclic
a) the abuser is commonly a friend or a relative antidepressants is:
b) physical indicators of sexual abuse include bruising a) peripheral neuropathy
b) photosensitivity
or pain in the genital region
c) children who have been abused often present with c) agranulocytosis
aggressive or anxious behaviors d) jaundice
d) Mmentally retarded or physically handicapped e) dry mouth
children may be at increased risk of abuse
e) local child welfare authorities must be notified in
cases of suspected child abuse, however there is no
duty to report child neglect

MCCQE 2002 Review Notes Sample Questions – 41


PSYCHIATRY . . . CONT.

27. A high risk of suicide is associated with which one of the 33. Which of the following symptoms of schizophrenia
following factors: responds best to anti-psychotic medication:
a) female aged less than 30 years a) asocial behavior
b) married male less than 30 years b) flat affect
c) public setting c) paranoid delusions
d) secondary gain from attempt d) lack of motivation
e) single male aged more than 60 years e) all of the above

28. A fixed unalterable belief that is false in its content and 34. A 53 year-old housewife presents with depression
in its social and cultural setting is called: marked by early morning wakening, diminished energy
a) an illusion and poor concentration. She is treated with supportive
b) a hallucination psychotherapy and imipramine 100 mg qhs.
c) a delusion After four weeks she is no better. You would next:
d) agnosia a) reassure her that antidepressants take 5-6 weeks to
e) paranoia be effective
b) begin intensive psychotherapy
29. Criteria for involuntary hospitalization of a suicidal c) switch to a serotonin reuptake inhibitor
patient include: d) increase the dose to 150 mg qhs
a) assessment done less than 72 hours prior to e) add lithium carbonate 1200 mg daily
completion of Form 1
b) presence of a mental disorder 35. Hypertensive encephalopathy is a serious complication of
c) acute risk of harm to themselves treatment with:
d) (a), (b), and (c) a) phenothiazines
e) (b) and (c) b) tricyclic antidepressants
c) lithium carbonate
30. Each of the following statements about affective d) MAOI antidepressants
disorders is true EXCEPT: e) benzodiazepines
a) patients with bipolar disorder show roughly the
same frequency of positive family history as do 36. Electroconvulsive therapy (ECT) is a first line treatment in
patients with unipolar disorder which one of the following conditions?
b) major depressive illness is more common in women a) obsessive compulsive disorder
than in men b) paranoid schizophrenia
c) bipolar affective disorder is far less common than c) generalized anxiety disorder
unipolar affective disorder d) acute mania
d) there is a different response to lithium in unipolar e) major depression with acute suicidal ideation
and bipolar disorder
e) imipramine is more likely to produce hypomania in 37. Tardive dyskinesia is:
bipolar patients than in unipolar patients a) an acute extrapyramidal side effect of phenothiazines
b) an acute anticholinergic side effect of tricyclic
31. A major depressive disorder is diagnosed only when the antidepressants
affective disturbance has existed for at least : c) a type of Parkinson’s disease
a) two weeks d) a side effect of excessive ECT treatments
b) four weeks e) a long term complication of chronic phenothiazine
c) six weeks administration
d) eight weeks
e) twelve weeks 38. Which of the following symptoms is most commonly found
in schizophrenia?
32. A 30 year-old man presents in emergency with right a) depressed mood
lower quadrant abdominal pain. His wife reports that he b) flight of ideas
had been drinking heavily in response to marital c) elevated mood
problems and had never had such pain before. d) delusional fears
Appendicitis was diagnosed and an appendectomy was e) thought insertion
successfully performed. Four days later the patient was
anxious, restless, unable to sleep and claimed his wife 39. Patients with conversion disorders will show each of the
was a stranger trying to harass him. The likeliest following EXCEPT:
diagnosis is: a) “La belle indifference”
a) paranoid reaction b) severe depression
b) delirium tremens c) loss of special sense function
c) mania d) secondary gains
d) schizophreniform reaction e) paralysis of voluntary muscles
e) post-operative delirium

42 – Sample Questions MCCQE 2002 Review Notes


PSYCHIATRY . . . CONT.

40. The syndrome of delirium tremens is associated with 48. Which of the following investigations need not be
each of the following EXCEPT: performed before starting treatment with lithium
a) rapidly fluctuating level of consciousness carbonate?
b) dehydration a) serum creatinine
b) serum electrolytes
c) visual hallucinations c) thyroid function studies
d) renal failure d) serum bilirubin
e) polyneuropathy e) all of the above

41. Which of the following is best treated with high dose 49. All of the following are common side effects of tricyclic
benzodiazepines: antidepressants EXCEPT:
a) schizophrenia, catatonic type a) dry mouth
b) major depression b) constipation
c) parkinsonian tremor
c) generalized anxiety disorder d) tachycardia
d) delirium tremens
e) psychogenic amnesia 50. The Theory of Behavioral Therapy employs which of the
following concepts:
42. Personality types predisposed to depression include a) reaction formation
which one of the following: b) reinforcement
a) dependent c) imprinting
b) antisocial d) autosuggestion
e) sublimation
c) schizoid
d) paranoid 51. Which of the following represents a contraindication for
e) schizotypal psychoanalysis?
a) the existence of transference feeling toward the
43. Which of the following neurological symptoms can be analyst
produced by antipsychotic drugs? b) the existence of countertransference feeling in the
a) akathesia analyst
b) shuffling gait c) the resistance to change by the patient
d) preoccupation with resolving a crisis situation
c) oculogyric crisis e) the existence of an underlying neurotic style of
d) tremor at rest behaviour
e) all of the above
52. Which of the following is least helpful in treating
44. Absolute contraindications to ECT include: depression?
a) pregnancy a) phenelzine
b) recent myocardial infarction b) imipramine
c) fractured pelvis c) lithium carbonate
d) lorazepam
d) brain tumor e) all of the above
e) all of the above
53. A young mother is very focused on the health of her
45. In the elderly delirium may be produced by the use of: 16 month-old. She keeps her house immaculate for fear
a) neuroleptics that dirt will harm her baby, she checks the lock on the
b) tricyclic antidepressants door at least ten times before retiring to bed, and she has
c) antiparkinsonian agents to get up and check that her child is still breathing at least
d) minor tranquilizers 3 times every night. She knows that her fears are irrational
but persists with these behaviors. The most likely
e) all of the above diagnosis is:
a) paranoid delusions not otherwise specified
46. Which of the following statements about simple phobia b) post-partum depression
is correct? c) obsessive compulsive disorder
a) it responds well to simple reassurance d) generalized anxiety disorder
b) it responds well to individual psychotherapy e) paranoid personality disorder
c) it responds well to benzodiazepines
d) it responds well to relaxation and desensitization 54. A thorough assessment for the presence/absence of
alcohol withdrawal should include questions about all of
e) all of the above the following EXCEPT:
a) nausea and vomiting
47. Toxicity due to lithium carbonate is associated with all b) mood
of the following EXCEPT: c) difficulty walking (ataxic gait)
a) nausea d) visual disturbances
b) serum lithium level of 0.1 mEq/L e) tremulousness
c) tremulousness
d) convulsions

MCCQE 2002 Review Notes Sample Questions – 43


PSYCHIATRY . . . CONT.

55. Which of the following will be least helpful with respect 63. Which of the following statements is true about paraphilias:
to distinguishing delirium from dementia: a) almost never diagnosed in women
a) disorientation at night b) treated with androgen drugs
b) duration of disorientation c) includes: exhibitionism, fetishm, voyeurism,
c) fluctuating level of consciousness pedophilia
d) mini mental status examination d) a and b
e) presence of visual hallucinations e) a and d
56. Onset of schizophrenia is typically in an affected 64. All of the following statements are correct EXCEPT:
individual’s: a) female orgasmic disorder can be caused by
a) early teens denervation of the lumbosacral spine
b) late teens/early 20s a) having a rigid, conservative sexual upbringing can
c) late 20s/early 30s result in delayed ejaculation in men
d) late 30s/early 40s a) phenothiazines are used to treat delayed ejaculation
e) late 40s/early 50s disorder in males
a) most delayed ejaculation is situational
57. Which of the following is an example of a negative a) most orgasmic disorders in women are psychological.
symptom of schizophrenia:
a) nihilistic delusion 65. Major Depressive Disorder with melancholic features is
b) flat affect most likely accompanied by:
c) auditory hallucinations a) echololia and /or echopraxia
d) catatonic behaviour b) hypersomnia
e) all of the above c) severe anhedonia
d) weight gain
58. Helpful treatments for post-traumatic stress disorder e) inappropriate posturing
symptoms include the following except:
a) SSRIs 66. Long acting antipsychotic medications:
b) relaxation techniques a) are available in both typical and atypical preparations
c) exposure therapy b) are administered IM or SC
d) cognitive restructuring c) are first line treatment for severe schizophrenia
e) all of the above may be helpful d) are less likely to cause EPS compared to the oral form
e) should be ideally started after the patient has had
59. The primary difference between factitious disorder and exposure to the oral form
malingering is:
a) factitious disorder involves the feigning of 67. A 55-year-old man with chronic schizophrenia is brought to
symptoms for external rewards (e.g. economic gain) the E.R. He is diaphoretic, pale and has a temperature of
b) factitious disorder involves the unintentional 41ºC, blood pressure of 170/100, and heart rate of 120. He
production of symptoms also exhibits muscle rigidity and a fluctuating level of
c) there is an absence of primary gain in factitious consciousness. The most diagnosis is:
disorder a) acute dystonic reaction
d) factitious disorder can involve only psychological b) tardive dyskinesia
symptoms c) neuroleptic malignant syndrome
e) there is an absence of secondary gain in factitious d) psychosis secondary to a general medical condition
disorder e) substance-induced psychosis
60. The following features tend to differentiate atypical 68. Obsessive-compulsive disorder is most consistent with:
antipsychotics from typical/conventional antipsychotics a) perfectionism, meticulous organization and
EXCEPT: cleanliness
a) atypicals are serotonin-dopamine antagonists a) the persistent need to shower because of the belief
b) atypicals tend to result in more specific dopamine that aliens are contaminating one’s body
blockade b) recurrent and intrusive thoughts that are inserted into
c) atypicals tend to result in lower levels of prolactin one’s mind by others
d) atypicals tend to cause fewer extrapyramidal side c) a persistent preoccupation with food and one’s
effects appearance accompanied by bingeing, purging and/or
e) atypicals tend to be more effective for the restricting behaviours
treatment of negative symptoms d) compulsively avoiding cracks in the side walk for fear
that if one does not, a relative might be subject to
61. Which of the following statements is true about sexual harm
dysfunction:
a) it is secondary to physical factors alone 69. Bipolar disorder may be characterized by:
b) it occurs only in females a) rapid cycling: at least one episode (manic or
c) the commonest cause is medications depressed) every 4 weeks
d) psychological factors account for 10% in women b) an increased risk in females
e) none of the above c) a lower risk in higher socioeconomic groups due to
decreased psychosocial stressors
62. Which of the following statements is true about d) hypomanic episodes without depressed episodes, as
premature ejaculation: in Bipolar II disorder
a) most common male sexual dysfunction e) none of the above
b) usually secondary to performance anxiety
c) has a 90% success rate with regards to treatment
d) treatment includes: excercises to focus on
experience vs performance, increasing stimulation
and control exercises
e) all of the above

44 – Sample Questions MCCQE 2002 Review Notes


RESPIROLOGY
1. which of the following is an incorrect statement regarding 6. All of the following but one are typical chest x-ray
transudative pleural effusions? findings in a person with COPD:
a) pleural LDH/serum LDH is > 0.6 a) hyperinflation
b) pleural protein/serum protein is < 0.5 b) flat hemidiaphragm
c) pelural LDH <2/3 upper limit of normal serum LDH c) increased AP diameter
d) all of the above d) increased retrosternal air space
e) none of the above e) increased peripheral markings

2. Which of the following organisms causes 7. Which of the following is NOT an indication of home
“walking pneumonia”? O2 in a person with COPD?
a) S. aureus a) PaO2 < 55 mmHg
b) S. pneumoniae b) PaO2 < 60 mmHg with erythrocytosis
c) H. fluenzae or cor pulmonale
d) Mycoplasma c) persistant hypoxemia after 3 weeks of
e) Legionella maximal therapy
d) PaO2 maintained > 80 mmHg during wakeful res
3. Which of the following is considered a positive tb skin increased by 250 ml during exercise or sleep
test?
a) induration of > 5 mm in close contact with an 8. Which of the following are associated with upper
HIV positive individual lobe fibrosis?
b) induration of < 5 mm in an adult with no risk factors a) farmer’s lung
c) induration of < 5 mm in a person with silicosis b) silicosis
d) induration of > 5 mm in a person with known TB c) sarcoid
exposure d) histiocytosis
e) X asbestos
4. The following are typical chest x-ray findings of a
malignant solitary nodule: 9. The differential diagnosis of obstructive lung disease
a) > 3 cm includes:
b) ill-defined margins a) COPD
c) “popcorn” pattern of calcification b) asthma
d) associated pleural effusions c) bronchiectasis
e) cavitation with thick wall d) CF
e) chest wall disease
5. All of the following but one are indicative of poorly
controlled asthma: 10. Which is NOT a feature of adenocarcinoma of the lung?
a) night time awakenings a) solitary peripheral pulmonary nodule
b) use of a beta-agonist 2x/week b) may appear as an alveolar infiltrate simulating
c) mild limitation of activities pneumonia
d) chronic dry cough c) frequently unilateral
d) metastasizes early

MCCQE 2002 Review Notes Sample Questions – 45


RHEUMATOLOGY
1. A 21 year-old bisexual man has a 4 week history of 6. A 70 year-old woman presents with acute knee arthritis.
intermittent diarrhea, urethral discharge, and pain in the Radiographs show meniscal calcification
right knee and left second toe. He has several oral (chondrocalcinosis). Analysis of the synovial fluid
ulcers, a clear urethral discharge, a scaly papular rash on reveals weakly positive birefringent rhomboid-shaped
palms and soles, onycholysis, sausage-like swelling of crystals. The crystals are most likely:
the left second toe, and heat and swelling of the right a) monosodium urate
knee. The results of Gram stains and cultures of urethral b) calcium hydroxyapatite
discharge are negative. Rheumatoid factor is not present. c) cholesterol
The most likely diagnosis is: d) calcium pyrophosphate dihydrate
a) Reiter’s syndrome e) dicalcium phosphate dihydrate (Brushite)
b) gonococcal arthritis
c) Behçet’s disease 7. Which of the following is not a disease modifying agent for
d) acquired immune deficiency syndrome rheumatoid arthritis?
e) psoriatic arthritis a) infliximab
b) gold
2. A 53 year-old presents to your office with pain and c) high dose corticosteroid
stiffness in both hands and knees of 6 months duration. d) methotrexate
Which of the following findings on your physical e) hydroxychloroquine
examination may help with a diagnosis:
a) joint tenderness/effusions 8. A 25-year-old male presents to the emergency room with a
b) maculopapular rash 6-week history of progressive weakness. On questioning,
c) iridocyclitis the man has difficulty combing his hair, brushing his teeth,
d) hepatosplenomegaly and climbing stairs. Night sweats, fever and malaise are
e) all of the above present. He has noticed a purplish swelling around his
eyes. Bloodwork shows a high AST and ALT. ALP is
3. Radiographic features of osteoarthritis of the knee normal. High on your differential diagnosis is:
include which of the following: a) rheumatoid arthritis
a) marginal erosions b) trauma-related back pain
b) juxta-articular osteopenia (demineralization) c) inflammatory myopathy
c) loss of articular cartilage with narrowing of the d) polyarteritis nodosa
radiologic joint space e) polymyalgia rheumatica
d) osteonecrosis (avascular necrosis) of the medial
femoral condyle 9. A 39-year-old female presents to your office with an
e) high riding patella (patella alta) insidious onset of right knee pain. The differential
diagnosis includes:
4. Which of the following is true about serologic testing in a) synovitis
SLE? b) degenerative arthritis
a) A positive ANA is specific for SLE c) ligament derangement
b) ds-DNA level correlates with disease activity in SLE d) bursitis
c) Anti-histone antibodies are seldom positive in e) all of the above
non-drug induced SLE
d) The majority of patients with SLE have anti-Sm 10. Systemic lupus erythematosus may be characterized by all
antibodies of the following EXCEPT:
e) Anti-Ro antibody is specific for SLE a) hemolytic anemia
b) discoid rash
5. The treatment of choice for thrombotic events in the c) serositis
antiphospholipid antibody syndrome is: d) erosive arthritis
a) intravenous steroids e) psychosis
b) high-dose oral steroids with a rapid taper
c) penicillamine 11. Physical examination manoeuvres specific to inflammatory
d) aspirin spondyloarthropathies include all of the following EXCEPT:
e) warfarin a) straight leg raising
b) occiput-wall distance
c) chest expansion
d) Schöeber’s test

46 – Sample Questions MCCQE 2002 Review Notes


RHEUMATOLOGY . . . CONT.

12. Cautions and contraindications of NSAIDs include: 19. First line pharmacologic treatment for osteoarthritis is:
a) a creatinine of 180 mmol/L a) acetaminophen 1000 mg po QID
b) peptic ulcer disease b) celecoxib 200 mg po BID
c) warfarin use c) glucosamine 500 mg po QID
d) cirrhosis d) ibuprofen 400 mg po TID
e) all of the above e) depo-medrol 20 mg intra-articularly

13. Small vessel vasculitides include all of the following 20. Which of the following medications is not associated with
EXCEPT: drug-induced lupus syndrome?
a) Wegener’s granulomatosis a) isoniazid
b) Churg-Strauss syndrome b) propranolol
c) essential cryoglobulinemia vasculitis c) hydralazine
d) Kawasaki’s disease d) cyclosporine
e) predominantly cutaneous vasculitis d) carbamazepine

14. A 30-year-old male presents with a history of mild upper 21. A 30-year-old woman comes to your office with a 1-day
respiratory tract infection progressing to a severe cough history of right knee pain and swelling. She has never had
and hemoptysis. Upon examination, you note a joint pain. She spent the day before working in her
nasoseptal perforation. Tests you should order include: garden, and was kneeling for most of the time. She has no
a) chest x-ray other associated symptoms, but has had several sexual
b) urine routine and microscopy partners within the last year. You aspirate her knee
c) CBC because you are worried about septic arthritis and send
d) c-ANCA requisitions for cell counts, microscopy, stat Gram stain,
e) all of the above glucose, LDH and culture. The lab phones you back in the
next hour with the following results:
15. A 50-year-old female presents with headache, scalp WBC 40,000
tenderness and painless loss of vision in her right eye. % PMN 35%
Your initial management is: Stat Gram stain negative
a) temporal artery biopsy Crystals negative
b) prednisone 1mg/kg Glucose 1.1mmol/L
c) infliximab infusion
d) cyclophsophamide 2mg/kg What is your next step?
e) consult to Ophthalmology a) Send the patient home and advise her to take extra-
strength acetaminophen - this is probably traumatic
16. Psoriatic arthritis may be characterized by: b) Advise her to go to the emergency room with a letter
a) subungual hyperkeratosis instructing the emergency staff to draw blood cultures
b) DIP involvement and start antibiotics
c) conjunctivitis c) Give the patient intra-articular steroids
d) dactylitis d) Send the patient home and ask her to return to your
e) all of the above office tomorrow
e) Refer her to a rheumatologist
17. All of the following statements about septic arthritis are
true EXCEPT:
a) permanent joint damage can quickly occur
b) N. gonorrhea accounts for 75% of cases in young
adults
c) Salmonella species characteristically infect those
with idiopathic thrombocytopenic purpura
d) predisposing factors include prior antibiotic use
e) IV antibiotics should be started empirically

18. Cauda equina syndrome is characterized by the following


EXCEPT:
a) saddle anesthesia
b) urinary retention
c) weak plantar flexion
d) decreased anal sphincter tone
e) posterior thigh pain

MCCQE 2002 Review Notes Sample Questions – 47


UROLOGY
1. An 84 year old man presents to your office complaining 5. A 55 year old man reluctantly admits to difficulty in
of a five day history of passing terminal bright red maintaining an erection sufficient for sexual intercourse
bloody for the past two years. He reports being healthy,
urine. He reports marked urgency and frequency but is however, he has not seen a physician in 10 years. Which
otherwise well and afebrile. In retrospect, he remembers of the following would not suggest a potential cause for
being quite ill with an infection in the recent past. his erectile dysfunction?
Analysis of a voided specimen shows 3 smooth and a) elevated blood pressure
rounded RBCs/hpf. Which of the following is NOT b) galactorhea
included in your differential diagnosis? c) acanthosis nigracans
a) benign prostatic hpertrophy (BPH) d) varicocele
b) post-streptococcal glomerulonephritis e) eruptive xanthoma
c) a stone lodged at the vesico-ureteric junction
d) a superficial bladder tumour 6. A 50 year old man presents with a 2 month history of
e) a coliform positive UTI irritative voiding symptoms and a fever. He initially saw
his family doctor who treated him for cystitis with
2. Of the following conditions, which is not considered a ciprofloxacin for 5 days. He now presents to you because
urological emergency? his symptoms have returned soon after he finished his
a) paraphimosis course of antibiotics. All of the following are likely
b) priapism TRUE except:
c) orchitis a) he never had a UTI
d) testicular torsion b) he might have complained of discomfort when
e) autonomic dysreflexia riding a bicycle
c) he might have also experienced back pain
3. A normal semen analysi includes all of the following d) this patient does not need to be investigated
EXCEPT: e) he could of had prostatitis and should have been
a) pH between 7.5-8.1 treated at least 3 weeks
b) a sperm count > 20 million sperm/ml
c) WBC < 10/hpf
d) motility > 50%
e) > 60% normal morphological forms

4. A mother brings her toilet-trained, 5 year old daughter


to see you with a 3 day history of dysuria, increased
frequency, and urgency incontinence. Her past medical
history is significant for a bag-specimen positive UTI
charaterized by high fever and vomiting at 2 years of age.
At the time, she was lost to follow up. Which of the
followin would be appropriate for your initial
management of this patient?
a) voiding cystourethrogram (VCUG)
b) a mid stream urine specimen
c) nuclear cystogram
d) DMSA renal scan
e) abdominal ultrasound

48 – Sample Questions MCCQE 2002 Review Notes


ANSWERS TO SAMPLE QUESTIONS
ANESTHESIA
1. B 2. B 3. D 4. C 5. B 6. A 7. B 8. E 9. C 10. C

11. A 12.D 13. C 14. C

CARDIAC AND VASCULAR SURGERY


1. 2. 3. 4. 5.

CARDIOLOGY
1. B 2. B 3. C 4. D 5. C 6. B 7. D 8. E 9. D 10. A

COMMUNITY HEALTH
1. B 2. D 3. D 4. A 5. D 6. D 7. E 8. A 9. B 10. A

11. B 12.E 13.E 14. A 15. C 16. D 17. C 18.E

DERMATOLOGY
1. 2. 3. 4. 5. 6. 7. 8. 9. 10.

11. 12. 13. 14. 15. 16. 17. 18. 19. 20.

21. D

EMERGENCY MEDICINE
1. C 2. C 3. B 4. D 5. D 6. E 7. E 8. B 9. C 10. B

11. B 12.B 13.B 14. E 15. D

ENDOCRINOLOGY
1. B 2. D 3. C 4. A 5. E 6. C 7. B 8. B 9. C 10. C

11. D 12.C 13.C 14. A 15. B 16. B 17. C 18.B 19. E 20. B

21. E 22.A

FAMILY MEDICINE
1. C 2. D 3. B 4. E 5. A 6. C 7. D 8. A 9. C 10. B

11. D 12.E 13. B 14. C

GASTROENTEROLOGY
1. B 2. A 3. D 4. C 5. B 6. C 7. E 8. B 9. C 10. D

11. A 12.B 13.C 14. A 15. A 16. B

MCCQE 2002 Review Notes Sample Questions – 49


ANSWERS TO SAMPLE QUESTIONS . . . CONT.

GERIATRIC MEDICINE
1. B 2. D 3. E 4. B 5. A

GYNECOLOGY
1. B 2. C 3. B, C, E, F, G, H, I 4. B, C, D, E, F, G, H 5. 1, B, C, D, E, G 6. B, C, F

7. A, B, E, G 8. B, C, G 9. A, E, F 10. A, B, C, D, F, G, H, I 11. E 12. A

13. D 14. E 15.E

HEMATOLOGY
1. B 2. D 3. E 4. B 5. A

INFECTIOUS DISEASES
1. D 2. A 3. C 4. D 5. B 6. E 7. B 8. D 9. C 10. E

11. B 12.C 13.B 14. A 15A

NEPHROLOGY
1. A 2. C 3. C 4. D 5. C 6. B 7. A 8. A 9. B 10. B

11. D 12.B 13.B 14. A 15. D

NEUROLOGY
1. B 2. D 3. A 4. C 5. C 6. D 7. A 8. E 9. C 10. C
11. E 12.D

NEUROSURGERY
1. B 2. D 3. B 4. A 5. C 6. D 7. B 8. A 9. B 10. C
11. D 12.D 13.C 14. B 15. A

OBSTETRICS
1. D 2. D 3. E 4. E 5. B 6. E 7. A 8. A 9. A 10. A
11. A 12.D 13.A 14. D 15. D

OPHTHALMOLOGY
1. B 2. A 3. B 4. C 5. D 6. B 7. D 8. A 9. C 10. C
11. A 12.C 13.D 14.B 15.A 16.C 17.D 18.C 19.B 20.C
21.A 22.C 23.B 24.E

ORTHOPEDICS
1. B 2. C 3. D 4. B 5. E

50 – Sample Questions MCCQE 2002 Review Notes


ANSWERS TO SAMPLE QUESTIONS . . . CONT.

OTOLARYNGOLOGY
1. C 2. B 3. C 4. E 5. B 6. B 7. D 8. C 9. C 10. A
11. B 12.B 13.C 14. D 15. C 16. A 17. B 18.D 19. C 20. B
21. C 22.C 23.B 24. D 25. B 26. E 27. C 28.D

PEDIATRICS
1. E 2. A 3. A 4. B 5. C 6. B 7. E 8. A 9. B 10. E
11. D 12.B 13.D 14. B 15. B 16. B 17. B 18.C 19. A 20. A
21. E 22.B 23.D 24. C 25. B 26. C 27. C 28.B 29. C 30. C
31. D 32.B 33.A 34. C 35. C 36. A 37. C 38.E 39. D 40. C
41. A 42.B 43.B 44. E 45. D 46. A 47. B 48.D 49. A 50. D
51. A 52.D 53.B 54.B 55. A 56. E 57. E 58.D

PHARMACOLOGY
1. C 2. C 3. C 4. D 5. C 6. E 7. B 8. D 9. E 10. D
11. 12.D

PLASTIC SURGERY
1. C 2. E 3. B 4. C 5. B 6. A 7. A 8. D 9. B 10. D
11. A 12.B

PSYCHIATRY
1. D 2. A 3. D 4. C 5. E 6. E 7. B 8. B 9. C 10. D
11. B 12.E 13.D 14. C 15. E 16. B 17. D 18.A 19. E 20. D
21. E 22.E 23.D 24. C 25. E 26. E 27. E 28.C 29. E 30. A
31. A 32.B 33. C 34. D 35. D 36. E 37. E 38.D 39. B 40. D
41. D 42.A 43.E 44. D 45. E 46. D 47. B 48.D 49. C 50. B
51. D 52.D 53.C 54. C 55. A 56. B 57. B 58.E 59. C 60. B
61. C 62.E 63.A 64. C 65. C 66. E 67. C 68.E 69. E

RESPIROLOGY
1. A 2. D 3. A 4. C 5. B 6. E 7. D 8. E 9. E 10. C

RHEUMATOLOGY
1. A 2. E 3. C 4. B 5. E 6. D 7. C 8. C 9. E 10. D
11. A 12.E 13.D 14. A 15. B 16. E 17. C 18.E 19. A 20. D
21. B

UROLOGY
1. B 2. C 3. A 4. A 5. D 6. D

MCCQE 2002 Review Notes Sample Questions – 51


KEY FEATURE QUESTIONS
Instructions for Examinees
“Key Feature” questions can pertain to solutions of clinical cases that involve data-gathering (e.g., history taking, physical examination, laboratory investi-
gations), diagnosis or treatment.
The Key Feature questions will ask for your answers in different ways: you may be asked to (i) select the correct response(s) from a menu of options, or
(ii) print the correct answer(s) on a blank line.
Please note the following guidelines before proceeding:
1. After reading the description of the case, read the question carefully. Take note as to whether you are told the maximum number of answers to
provide. If you exceed this number, you will receive no credit for the question. If you select or list fewer answers, you will still receive credit
for any of your answers which are correct.
2. If you are asked to select your answers from a menu:
a. Read the complete menu of options before recording your answers.
b. Make certain that the number of answers you record is not greater than the number you have been asked to select.
3. If you are asked to list your answers below the question:
a. Ensure that you print each answer legibly.
b. Record each answer on a separate blank line.
c. Do not record more than the maximum number you are asked to list.

CASE1
A 30 year-old homeless male presents with a 4 week history of feeling
generally unwell with anorexia, weight loss and intermittent fever. He
lives primarily on the streets or occasionally in a men’s shelter. He admits
to regular use of intravenous heroin for the last seven years and often
shares needles.

QUESTION 1 (CASE 1) QUESTION 2 (CASE 1)


Given this history, which of the following diagnoses would you consider? A mitral regurgitant murmur is auscultated in this patient.
Select up to five. His temperature is 39.0ºC.
1. Acute pancreatitis Which of the following other findings may be found on physical examina-
2. AIDS related complex tion given the diagnoses being considered?
3. Aspergillosis Select up to five.
4. Bacterial endocarditis 1. Ascites
5. Chronic pancreatitis 2. Campbell De Morgan spots
6. Delirium tremens 3. Cannon “a” waves
7. Diabetes mellitus 4. Cyanosis
8. Hepatitis B 5. Erythema nodosum
9. Hepatocellular carcinoma 6. Follicular keratosis
10. Histoplasmosis 7. Gynecomastia
11. Leukemia 8. Hepatomegaly
12. Lymphoma 9. Janeway lesions
13. Osteomyelitis 10. Jaundice
14. Peptic ulcer disease 11. Koilonychia
15. Pericarditis 12. Livedo reticularis
16. Pyelonephritis 13. Osler nodes
17. Renal failure 14. Pallor
18. Secondary syphilis 15. Palmar erythema
19. Talc pneumonitis 16. Papilledema
20. Tuberculosis 17. Purpura
18. Roth spots
19. Spider nevi
20. Splenomegaly

52 – Sample Questions MCCQE 2002 Review Notes


KEY FEATURE QUESTIONS . . . CONT.

CASE 2 CASE 3
A babysitter brings a 14-month-old to emergency because of incon- An 84 year-old female is brought to the Emergency Department by her
solable crying. The child appears well cared for and healthy but a daughter who states that her mother has not been herself over the past
"bruise" is noted on the left humerus and a well-circumscribed small three days. Further questioning reveals that the mother has been less
fresh burn is noted on the other shoulder. attentive than usual and more withdrawn, her conversation is often
rambling and she has not been sleeping well. Prior to this, she functioned
QUESTION 3 (CASE 2) very well.
List four well recognized risk factors for child abuse: QUESTION 6 (CASE 3)
1. _____________________________________________________________
What diagnoses would you consider at this time? List up to two.
2. _____________________________________________________________
1. _____________________________________________________________
3. _____________________________________________________________
2. _____________________________________________________________
4. _____________________________________________________________
QUESTION 7 (CASE 3)
QUESTION 4 (CASE 2)
What additional aspects of history would be most important to know?
Which of these physical findings are most suggestive of child abuse? Select up to four.
Select up to four. 1. Alcohol use
1. Alopecia 2. Complete psychiatric history
2. Blue sclera 3. Family history of Alzheimer’s disease
3. Bruises on shins and elbows 4. Financial status
4. Buttocks wasting 5. History of fever
5. Caput medusa 6. History of incontinence
6. Clubbing 7. History of osteoarthritis
7. Condylomata acuminata 8. History of Parkinson’s disease
8. Eczema 9. History of previous CVA
9. Full fontanelle 10. Immunization status
10. Healed laceration on chin 11. Marital status
11. Hutchison teeth 12. Medication history
12. Impetigo 13. Occupational history
13. Large purple macular lesion on buttock 14. Pattern of symptoms over the course of the day, e.g. fluctuations
14. Limp 15. Presence of hallucinations
15. Oral thrush 16. Previous history of depression
16. Petechiae 17. Previous seizure disorder
17. Pitted finger nails 18. Previous surgery
18. Proptosis 19. Recent travel
19. Retinal hemorrhage 20. Smoking history
20. Seborrhea
QUESTION 8 (CASE 3)
QUESTION 5 (CASE 2)
The interview confirms the daughter's description of the mother. Physical
You suspect this is a case of child abuse. Which of the following are exam reveals: BP 120/80 supine and 100/70 sitting, RR=22, P=104,
appropriate steps in the initial management. Select up to four. T=38.7ºC. JVP is at the sternal angle. Respiratory exam reveals dullness,
1. Abdominal ultrasound increased tactile fremitus, crackles and bronchial breath sounds all in the
2. Ascorbic acid level left base. Heart sounds are normal. The abdominal exam is normal. CNS
3. Bone density studies exam does not reveal any focal findings. What would you include in your
4. CBC initial investigations? Select up to four.
5. Chest x-ray 1. ALT
6. Coagulation studies 2. AST
7. Creatinine 3. Blood cultures
8. Dietary history 4. B12
9. ECG 5. CBC
10. Factor VIII assay 6. Chest x-ray
11. Liver/spleen scan 7. CPK
12. MRI 8. CT scan of head
13. Notify CAS (Children’s Aid Society) 9. Drug screen
14. Notify police 10. EEG
15. Notify public health nurse 11. Electrolytes
16. Ophthalmology consult 12. HIV test
17. Psychiatry consult 13. MRI of head
18. Serum calcium, phosphorus and alkaline phosphorus 14. Protein electrophoresis
19. Skeletal survey 15. PT and PTT
20. Urinalysis 16. RBC folate
17. SPECT scan
18. TSH
19. Urea and creatinine
20. VDRL

MCCQE 2002 Review Notes Sample Questions – 53


KEY FEATURE QUESTIONS . . . CONT.

CASE 4 CASE 5
A 57 year-old man presents to the Emergency Department with a 12 hour A 30 year-old woman comes to your office with a 3 day history of vulvar
history of left flank pain. and vaginal pruritus.
QUESTION 12 (CASE 5)
QUESTION 9 (CASE 4)
What diagnosis would you consider at this time? List up to three.
What diagnoses would you consider at this time?
Select up to four. 1. _____________________________________________________________
1. Ruptured abdominal aortic aneurysm
2. Bowel obstruction 2. _____________________________________________________________
3. Appendicitis
4. Pyelonephritis 3. _____________________________________________________________
5. Diverticulitis
6. Renal calculi QUESTION 13 (CASE 5)
7. Biliary colic
8. Renal cell carcinoma What features on the history would you inquire about to make a diagno-
9. Pneumonia sis? Choose up to seven.
1. Date of last menstrual period
2. Sexual history
QUESTION 10 (CASE 4) 3. Vaginal discharge
4. Vulvar erythema
On history and physical exam, what symptoms and signs might aid in 5. Vaginal odor
diagnosis? Choose up to seven. 6. Recent antibiotic therapy
1. Fever/chills 7. Diabetes
2. Weight loss 8. Macular rash
3. Rebound tenderness 9. Psychiatric history
4. Costovertebral angle tenderness 10. Headache
5. Urgency and frequency 11. Allergies
6. Pruritus 12. Dysuria
7. Hypotension 13. Use of douches
8. Pulsatile abdominal mass 14. Sore throat
9 Nausea and vomiting 15. Previous history of STDs
10. Left flank mass
11. Productive cough QUESTION 14 (CASE 5)
12. Abdominal guarding
13. Pale stools and dark urine What tests would you do to confirm the diagnosis? Choose up to four.
14. Hematuria 1. Urine C&S
2. Wet prep of vaginal discharge for microscopy
3. Urinalysis
QUESTION 11 (CASE 4) 4. CBC
5. ESR
What tests might be necessary to confirm your diagnosis? 6. Culture for gonorrhea
Choose up to six. 7. Culture for chlamydia
1. ESR 8. Abdominal CT
2. Serum amylase 9. Pelvic ultrasound
3. Urine R&M 10. Culture for Trichomonas vaginalis
4. Chest x-ray
5. Abdominal/pelvic ultrasound
6. 3 views of the abdomen
7. Urine C&S
8. Abdominal CT CASE 6
9. IVP
10. Total bilirubin A 65 year-old man in the emergency department has RUQ pain. He is
11. Electrolytes otherwise healthy.
12. 24 hour creatinine clearance
13. KUB QUESTION 15 (CASE 6)
14. Serum Mg
15. Serum alkaline phosphatase List three diagnoses you should consider in this patient.
16. Barium enema
17. Serum Ca 1. _____________________________________________________________
18. CBC
2. _____________________________________________________________
3. _____________________________________________________________

54 – Sample Questions MCCQE 2002 Review Notes


KEY FEATURE QUESTIONS . . . CONT.

QUESTION 16 (CASE 6) QUESTION 20 (CASE 8)


What are the most important questions to ask about on history? With respect to the diagnosis, which elements of the history and physi-
Choose up to seven. cal will be most important in determining the etiology of this baby’s dis-
1. Pruritus tress? Select up to four.
2. Pale stools 1. Delivery mode (C-section, vaginal)
3. Fever/chills 2. Feeding pattern
4. Hematuria 3. Grunting
5. Jaundice 4. Head circumference
6. Anorexia 5. Heart sounds
7. Dark urine 6. History of maternal diabetes
8. Weight loss 7. Indrawing
9. Back pain 8. Maternal temperature
10. Diarrhea 9. Meconium stained amniotic fluid
11. Alcohol/drug history 10. Oxygen requirement
12. Nausea and vomiting 11. Presence of bruising
13. Fatigue 12. Presenting part (breech, vertex)
14. Cold intolerance 13. Previous obstetric history
14. Tachypnea
15. Vital signs
CASE 7
A 60 year-old woman comes to your office because she has seen bright CASE 9
red blood in the toilet bowl after bowel movements for the past month.
She is otherwise healthy. A 45 year-old migrant farm worker comes to see you with sudden onset
of severe pain in his back and right leg after lifting some bales of hay.
The pain radiates into the lateral aspect of the right foot and is associat-
QUESTION 17 (CASE 7) ed with numbness along the lateral aspect of the foot. He has had a 5
year history of lower back pain with a similar episode of pain 1 year ago
What are three common diagnoses you should consider? which was concentrated at the posterolateral aspect of the right calf
and lateral aspect of the right foot. It became progressively worse,
1. _____________________________________________________________ then resolved over a 3 week period.
2. _____________________________________________________________
QUESTION 21 (CASE 9)
3. _____________________________________________________________
What elements of the history would you ask about in order to formulate
your diagnosis? Choose up to five.
QUESTION 18 (CASE 7) 1. Diffuse muscle cramps
2. Alcohol intake
What questions would you ask on history to help determine the cause of 3. Effect of NSAIDs
the bleeding? Choose up to five. 4. Family history of back problems
5. Previous history of back injury
1. Family history of inflammatory bowel disease 6. Shoulder pain
2. Hematemesis 7. Weakness in right leg
3. Recent weight loss 8. Allergies
4. Nausea and vomiting 9. Knee problems
5. Change in bowel habits 10. Urinary incontinence
6. Abdominal pain 11. Skin rashes
7. Fever/chills 12. Previous effective physiotherapy
8. Family history of colo-rectal cancer 13. Headaches
9. Blood mixed with stool or coating the surface 14. Nocturia
10. Past history of hemorrhoids
11. Dietary history
12. Smoking history QUESTION 22 (CASE 9)
13. Pain on defecation
14. Allergies What elements of the physical exam would you focus on?
Choose up to four.
1. Examination of skin
2. Range of motion of lumbar spine
3. Blood pressure
CASE 8 4. Inspection of muscle bulk in lower extremities
5. Sensory exam of arms
You are called to the nursery to assess a 2 hour old term newborn with 6. Peripheral pulses
respiratory distress. The only information the nurse gives you over the 7. Reflexes in lower extremities
phone is that the baby was born this morning, had good Apgars and 8. Motor testing of legs
arrived in the nursery pink on room air and in no distress. 9. Motor testing of arms
10. Sensory exam of legs
QUESTION 19 (CASE 8) 11. Bowstring sign
12. Respiratory rate
What are the most common diagnoses that would be in your differential 13. Temperature
for this infant? List up to three. 14. Straight leg raise test
15. Abdominal exam
1. _____________________________________________________________ 16. Rectal exam
17. Pulse
18. Range of motion of knee
2. _____________________________________________________________ 19. Auscultation of chest
20. Romberg test
3. _____________________________________________________________
MCCQE 2002 Review Notes Sample Questions – 55
KEY FEATURE SCORING GUIDE
QUESTION 1, CASE 1 QUESTION 5, CASE 2
1. Maximum Number to be selected: = 5 1. Maximum Number to be selected: = 4

2. The Number you selected (S): = ___ 2. The Number you selected (S): = ___

3. Is “S” greater than the Maximum? YES Your Score = 0. 3. Is “S” greater than the Maximum? YES Your Score = 0.
NO Continue. NO Continue.

4. There are 5 correct answers: 2, 4, 8, 13, 20 4. There are 4 correct answers: 6, 13, 16, 19

5. Number of correct answers you selected: = ___ 5. Number of correct answers you selected: = ___

QUESTION 2, CASE 1 QUESTION 6, CASE 3


1. Maximum Number to be selected: = 5 1. Maximum Number to be listed: = 2

2. The Number you selected (S): = ___ 2. The Number you listed (L): = ___

3. Is “S” greater than the Maximum? YES Your Score = 0. 3. Is “L” greater than the Maximum? YES Your Score = 0.
NO Continue. NO Continue.

4. There are 5 correct answers: 9, 13, 14, 18, 20 4. There are 2 correct answers:

5. Number of correct answers you selected: = ___ 1. Delirium


2. Acute Confusional State

5. Number of correct answers you listed: = ___


QUESTION 3, CASE 2
1. Maximum Number to be listed: = 4
QUESTION 7, CASE 3
2. The Number you listed (L): = ___
1. Maximum Number to be selected: = 4
3. Is “L” greater than the Maximum? YES Your Score = 0.
NO Continue. 2. The Number you selected (S): = ___

4. There are 8 correct answers: 3. Is “S” greater than the Maximum? YES Your Score = 0.
NO Continue.
1. Prematurity
2. Difficult child 4. There are 5 correct answers: 1, 5, 12, 14, 15
3. History of parental child abuse
4. History of substance abuse 5. Number of correct answers you selected: = ___
5. Social isolation
6. Developmental delay
7. Poverty
8. Parental character/personality disorder QUESTION 8, CASE 3
5. Number of correct answers you listed: = ___ 1. Maximum Number to be selected: = 4

2. The Number you selected (S): = ___

QUESTION 4, CASE 2 3. Is “S” greater than the Maximum? YES Your Score = 0.
NO Continue.
1. Maximum Number to be selected: = 4
4. There are 5 correct answers: 3, 5, 6, 11, 19
2. The Number you selected (S): = ___
5. Number of correct answers you selected: = ___
3. Is “S” greater than the Maximum? YES Your Score = 0.
NO Continue.

4. There are 4 correct answers: 7, 9, 14, 19

5. Number of correct answers you selected: = ___

56 – Sample Questions MCCQE 2002 Review Notes


KEY FEATURE SCORING GUIDE . . . CONT.

QUESTION 9, CASE 4 QUESTION 13, CASE 5


1. Maximum Number to be selected: = 4 1. Maximum Number to be selected: = 7

2. The Number you selected (S): = ___ 2. The Number you selected (S): = ___

3. Is “S” greater than the Maximum? YES Your Score = 0. 3. Is “S” greater than the Maximum? YES Your Score = 0.
NO Continue. NO Continue.

4. There are 4 correct answers: 1, 4, 6, 8 4. There are 8 correct answers: 2, 3, 5, 6, 7, 12,


13, 15
5. Number of correct answers you selected: = ___
5. Number of correct answers you selected: = ___

QUESTION 10, CASE 4


QUESTION 14, CASE 5
1. Maximum Number to be selected: = 7
1. Maximum Number to be selected: = 4
2. The Number you selected (S): = ___
2. The Number you selected (S): = ___
3. Is “S” greater than the Maximum? YES Your Score = 0.
NO Continue. 3. Is “S” greater than the Maximum? YES Your Score = 0.
NO Continue.
4. There are 8 correct answers: 1, 2, 4, 5, 7, 8,
10, 14 4. There are 4 correct answers: 2, 6, 7, 10

5. Number of correct answers you selected: = ___ 5. Number of correct answers you listed: = ___

QUESTION 11, CASE 4 QUESTION 15, CASE 6


1. Maximum Number to be selected: = 6 1. Maximum Number to be listed: = 3

2. The Number you selected (S): = ___ 2. The Number you listed (L): = ___

3. Is “S” greater than the Maximum? YES Your Score = 0. 3. Is “L” greater than the Maximum? YES Your Score = 0.
NO Continue. NO Continue.

4. There are 7 correct answers: 3, 5, 7, 8, 9, 13, 4. There are 3 correct answers:


18
1. Biliary colic
5. Number of correct answers you selected: = ___ 2. Acute cholecystitis
3. Bile duct carcinoma

5. Number of correct answers you listed: = ___


QUESTION 12, CASE 5
1. Maximum number to be selected = 3
QUESTION 16, CASE 6
2. The number you listed (L): = ___
1. Maximum Number to be selected: = 7
3. Is “L” greater than the Maximum? YES Your Score = 0.
NO Continue. 2. The Number you selected (S): = ___

4. There are 4 possible answers: 3. Is “S” greater than the Maximum? YES Your Score = 0.
NO Continue.
1. Bacterial vaginosis
2. Vaginal/vulvar candidiasis 4. There are 7 correct answers: 1, 2, 3, 5, 6, 7, 8
3. Trichomonas vaginalis
4. Chemical vaginitis 5. Number of correct answers you selected: = ___

5. Number of correct answers you listed: = ___

MCCQE 2002 Review Notes Sample Questions – 57


KEY FEATURE SCORING GUIDE . . . CONT.

QUESTION 17, CASE 7 QUESTION 20, CASE 8


1. Maximum Number to be listed: = 3 1. Maximum Number to be selected: = 4

2. The Number you listed (L): = ___ 2. The Number you selected (S): = ___

3. Is “L” greater than the Maximum? YES Your Score = 0. 3. Is “S” greater than the Maximum? YES Your Score = 0.
NO Continue. NO Continue.

4. There are 4 correct answers: 4. There are 4 correct answers: 1, 6, 8, 9

1. Hemorrhoids 5. Number of correct answers you selected: = ___


2. Anal fissure
3. Colon or rectal cancer
4. Proctitis or IBD or colitis
QUESTION 21, CASE 9
5. Number of correct answers you listed: = ___
1. Maximum Number to be selected: = 5

2. The Number you selected (S): = ___


QUESTION 18, CASE 7
3. Is “S” greater than the Maximum? YES Your Score = 0.
1. Maximum Number to be selected: = 5 NO Continue.

2. The Number you selected (S): = ___ 4. Essential answers are: 7 and 10

3. Is “S” greater than the Maximum? YES Your Score = 0. 5. Did you select both essential answers?
NO Continue. NO Your Score = 0.
YES Continue.
4. There are 6 correct answers: 1, 5, 8, 9, 10, 13
6. There are 5 correct answers: 3, 5, 7, 10, 12
5. Number of correct answers you selected: = ___
7. Number of correct answers you listed: = ___

QUESTION 19, CASE 8


QUESTION 22, CASE 9
1. Maximum Number to be listed: = 3
1. Maximum Number to be selected: = 6
2. The Number you listed (L): = ___
2. The Number you selected (S): = ___
3. Is “L” greater than the Maximum? YES Your Score = 0.
NO Continue. 3. Is “S” greater than the Maximum? YES Your Score = 0.
NO Continue.
4. There are 4 correct answers:
4. There are 8 correct answers: 2, 4, 7, 8, 10, 11,
1. Transient tachypnea of the newborn or wet lung 14, 16
2. Meconium aspiration
3. Pneumonia 5. Number of correct answers you selected: = ___
4. Cardiac disease

5. Number of correct answers you listed: = ___

58 – Sample Questions MCCQE 2002 Review Notes

You might also like